You are on page 1of 88

7 PLANE GEOMETRY-I

For IOQM

INTRODUCTION TO EUCLID GEOMETRY


The credit for introducing geometrical concepts goes to the distinguished Greek mathematician 'Euclid'
who is known as the "Father of Geometry" and the word 'geometry' comes from the Greek words
'geo' which means 'Earth' and 'metreon' which means 'measure'.

(a) Euclid’s definitions


(i) A point is that which has no part.
(ii) A line is breadthless length.
(iii) The ends of a line segment are points.
(iv) A straight line is that which has length only.
(v) A surface is that which has length and breadth only.
(vi) The edges of a surface are lines.
(vii) A plane surface is that which lies evenly with the straight lines on itself.

(b) Some Undefined terms


There are three basic concept in geometry, namely, ’’point” , ”line” and “plane”. It is not possible to
define these three concepts precisely. We can, however have a good idea of these concept by
considering examples given below
Point: A point is represented by a fine dot mae by sharp pencilon a sheet of paper. A Point is denoted
by capital letter A, B, P, Q, R etc.
Plane: The surface of a smooth wall or the surface of a sheet of paper or the surface of a smooth black
board are close example of a plane.
Line: If we fold a piece of paper, the crease in the paper represents a geometrical straight line. The edge
of ruler, the edge of the top of a table, the meeting place of two walls of a room are close examples of a
geometrical straight line.

(c) Some Geometrical concepts


Axioms: The basic facts which are taken for granted, without proof, are called axioms.
Ex. Halves of equal are equal.
Statements: A sentence which can be judged to be true or false is called a sentence.
Ex. The sum of the angles of a triangle is 180º, is a true statement.
Theorems: A statement that requires a proof, is called a theorem. Estabilishing the truth of a theorem is
known as proving the theorem.
Ex. The sum of all angles around a point is 360º
Corollary: A statement whose truth can easily be decided from a theorem, is called its corollary.

PW OLYMPIAD WALLAH 1
IOQM
Conjecture: In mathematics, a conjecture is a conclusion or proposition based on incomplete
information, for which no proof has been found. Ex. Goldbach Conjecture state that every even
integer greater than two is the sum of two prime numbers.

(d) Some terms related to geometry


Point: A point is an exact location. A fine dot represent a point. Point is denoted by capital letter A, B,
P, Q, R etc.
Line segment: The straight path between two points A and B is called the line segment AB . The point
A and B are called the end points of the line segment AB . A line segment has a definite length.
Line: A line segment AB when extended indefinitely in both direction is called the line AB . A line
has no end points. A line has no definite length. Some time, we label lines by small letters l, m, n etc.

Interior point of a line segment: A point R is called an interior point of a line segment PQ if R lies
between P and Q but R is neither P nor Q.

Congruence of line segment: Two line segments AB and CD are congruent if trace copy of one can be
superposed on the other so as to cover it completely and exactly in this case we write AB  CD. In
other words we can say two lines are congruent if their lengths is same.

Distance between two points: The distance between two points P and Q is the length of line segment PQ

Ray: Directed line segment is called a ray. If AB is a ray then it is denoted by AB . Point A is called
initial point of ray.

Opposite rays: Two rays AB and AC are said to be opposite rays if they are collinear and point A is
the only common point of the two rays.

Collinear points: Three or more points are said to be collinear if there is a line which contains all of them.

Concurrent Lines: Three or more lines are said to be concurrent if there is a point which lies on all of them.

PW OLYMPIAD WALLAH 2
IOQM
Intersecting lines: Two lines are intersecting if they have a common point. The common point is called
the point of intersection.

Parallel lines: Two lines l and min a plane are said to be parallel lines if they do not have a common
point.

Plane: A plane is a surface such that every point of the line joining any two points on it lies on it.

(e) Euclid’s Axioms


(i) Things which are equal to the same thing are equal to one another.
(ii) If equal are added to equals, the wholes are equal.
(iii) If equal are subtracted from equals, the remainders are equal.
(iv) The things which coincide with one another are equal to one another.
(v) The whole is greater than the part.
(vi) The thing which are double of the same thing are equal to one another.
(vii) The thing which are halves of the same thing are equal to one another.

(f) Euclid’s Five Postulates


(i) A straight line may be drawn from any one point to any other point.
(ii) A terminated line or a line segment can be produced infinitely.

(iii) A circle can be drawn with any centre and of any radius.
(iv) All right angles are equal to one another.
(v) If a straight line falling on two straight lines makes the interior angles on the same side of it taken
together less than two right angles, then the two straight lines if produced infinitely meet on that
side on which the sum of angles are less than two right angles.
(g) Incidence Axioms
(i) A line is the collection of infinite number of points.
(ii) Through a given point, an infinite lines can be drawn.

(iii) Given two distinct points, there is one and only one line that contains both the points.

PW OLYMPIAD WALLAH 3
IOQM
(h) Parallel Axioms
If P is a point outside a line , then one and only one line can be drawn through P which is parallel to .

The above axioms may also be re stated as follows:”Two intersecting lines cannot both be parallel to
the same line’”.
(i) Some more results
(i) Two distinct lines can not have more than one point in common.

(ii) Two lines which are both parallel to the same line, are parallel to each other.
i.e.  || n, m || n   || m

Solved Examples

1
Example : If a point C lies between two points A and B such that AC = BC, then prove that AC = AB.
2
Explain by drawing the figure.
Solution : According to the given statement, the figure will be as shown alongside in which the point C lies
between two points A and B such that AC = BC.
Clearly, AC + BC = AB
 AC + AC = AB [ AC = BC]
 2AC = AB

1
And, AC = AB
2
Example : Give a definition for each of the following terms. Are there other terms that need to be defined first?
What are they, and how might you define them?
(i) parallel lines (ii) perpendicular lines
(iii) line segment (iv) radius
Solution : (i) Parallel lines: Lines which don’t intersect any where are called parallel lines.
(ii) Perpendicular lines: Two lines which are at a right angle to each other are called perpendicular
lines.
(iii) Line segment: It is a terminated line.
(iv) Radius: The length of the line-segment joining the centre of a circle to any point on its
circumference is called its radius.

PW OLYMPIAD WALLAH 4
IOQM
Example : How would you rewrite Euclid's fifth postulate so that it would be easier to understand?
Solution : Two distinct intersecting lines cannot be parallel to the same line.

Example : Does Euclid's fifth postulate imply the existence of parallel lines? Explain.
Solution : If a straight line  falls on two straight lines m and n such that sum of the interior angles on one side

of  is two right angles, then by Euclid's fifth postulate the line will not meet on this side of . Next,

we know that the sum of the interior angles on the other side of line also be two right angles.
Therefore they will not meet on the other side. So, the lines m and n never meet and are, therefore
parallel.
Theorem: If , m, n are lines in the same plane such that  intersects m and n || m, then  intersects
n also.
Given: Three lines , m, n in the same plane s.t.  intersects m and n || m.

To prove: Lines and n are intersecting lines.

Proof: Let  and n be non intersecting lines. Then,  || n.


But, n || m [Given]
  || n and n || m   || m

  and m are non-intersecting lines.

This is a contradiction to the hypothesis that  and m are intersecting lines.


So our supposition is wrong.
Hence,  intersects line n.

Example : If lines AB, AC, AD and AE are parallel to a line , then prove that points A, B, C, D and E are
collinear.
Solution : Lines AB, AC, AD and AE are parallel to a line .
To prove: A, B , C, D, E are collinear.
Proof: Since AB, AC, AD and AE are all parallel to a line . Therefore point A is outside  and
lines AB, AC, AD,
AE are drawn through A and each line is parallel to .
But by parallel lines axiom, one and only one line can be drawn through the point A outside it and
parallel to .
This is possible only when A, B, C, D and E all lie on the same line. Hence, A, B, C, D and E are
collinear.

PW OLYMPIAD WALLAH 5
IOQM
LINE

A line has length but no width and no thickness.

ANGLE
An angle is the union of two non–collinear rays with a common initial point. The common initial point
is called the ‘vertex’ of the angle and two rays are called the ‘arms’ of the angles.

❖ REMARK:
Every angle has a measure and unit of measurement is degree.
One right angle = 90º
1º = 60’ (minutes)
1’ = 60” (Seconds)
Angle addition axiom: If X is a point in the interior of BAC, then m BAC = m BAX + m XAC.

(a) Types of Angles:


(i) Right angle: An angle whose measure is 90º is called a right angle.

(ii) Acute angle: An angle whose measure is less than 90º is called an acute angle.

0° < BOA < 900

PW OLYMPIAD WALLAH 6
IOQM
(iii) Obtuse angle: An angle whose measure is more than 90º but less than 180º is called an obtuse angle.

90º < AOB < 180º.

(iv) Straight angle: An angle whose measure is 180º is called a straight angle.

(v) Reflex angle: An angle whose measure is more than 180º is called a reflex angle.

180º < AOB < 360º.

(vi) Complementary angles: Two angles, the sum of whose measures is 90º are called
complementary angles.

AOC & BOC are complementary as their sum is 90º.

(vii) Supplementary angles: Two angles, the sum of whose measures is 180º, are called the
supplementary angles.

AOC & BOC are supplementary as their sum is 180º.

(viii) Angle Bisectors: A ray OX is said to be the bisector of AOB, if X is a point in the interior of
AOB, and AOX = BOX.

PW OLYMPIAD WALLAH 7
IOQM
(ix) Adjacent angles: Two angles are called adjacent angles, if
(A) they have the same vertex,
(B) they have a common arm,
(C) non common arms are on either side of the common arm.

AOX and BOX are adjacent angles, OX is common arm, OA and OB are non common arms
and lies on either side of OX.

(x) Linear pair of angles: Two adjacent angles are said to form a linear pair of angles, if their non
common arms are two opposite rays.

AOC + BOC = 180º.

(xi) Vertically opposite angles: Two angles are called a pair of vertically opposite angles, if their arms
form two pairs of opposite rays.

 AOC &  BOD form a pair of vertically opposite angles. Also  AOD &  BOC form a pair of
vertically opposite angles.
If two lines intersect, then the vertically opposite angles are equal i.e. AOC = BOD and BOC =
AOD.
(b) Angles Made by a Transversal with two Parallel Lines:
Transversal: A line which intersects two or more given parallel lines at distinct points is called a
transversal of the given lines.

PW OLYMPIAD WALLAH 8
IOQM
(i) Corresponding angles: Two angles on the same side of a transversal are known as the
corresponding angles if both lie either above the two lines or below the two lines, in figure  1 &
 5,  4 &  8,  2 &  6,  3 &  7 are the pairs of corresponding angles.
If a transversal intersects two parallel lines then the corresponding angles are equal i.e.  1 =  5,
 4 =  8,  2 =  6 and  3 =  7.

(ii) Alternate interior angles:  3 &  5,  2 &  8, are the pairs of alternate interior angles. If a
transversal intersects two parallel lines then the each pair of alternate interior angles are equal i.e.
 3 =  5 and  2 =  8.

(iii) Co– interior angles: The pair of interior angles on the same side of the transversal are called pairs
of consecutive or co – interior angles. In figure  2 &  5,  3 &  8, are the pairs of co–interior
angles.
If a transversal intersects two parallel lines then each pair of consecutive interior angles are
supplementary i.e.  2 +  5 = 180º and  3 +  8 = 180º.

Solved Examples

Example : Find the measure of an angle, if six times its complement is 12º less than twice its supplement.
Solution : Let the measure of the required angle be xº.
Then, measure of its complement = (90 – x)º.
Measure of its supplement = (180 – x)º.
 6 (90 – x) = 2 (180 – x) – 12
 540 – 6x = 360 – 2x – 12
 4x = 192  x = 48º.
Hence, the required angle is 48º.

Example : In the adjoining figure, ABC = 100º, EDC = 120º and AB || DE. Then, find BCD.

Solution :

Produce AB to meet CD at F.
BFD = EDF = 120° [alternate interior s]
 BFC = (180° – 120°) = 60°
 CBF = (180° – 100°) = 80°
 BCF = 180° – (60° + 80°) = 40°.
PW OLYMPIAD WALLAH 9
IOQM
POLYGON
A closed plane figure bounded by line segments is called a polygon.
A polygon is named according to the number of sides it has:

In general, a polygon having n sides is called 'n' sided polygon.

❖ Diagonal of Polygon:
Line segment joining any two non–consecutive vertices of a polygon is called its diagonal.

❖ Convex Polygon:
If all the interior angles of a polygon are less than 1800, it is called a convex polygon.

❖ Concave Polygon:
If one or more of the interior angles of a polygon is greater than 1800 i.e. reflex, it is called a concave
polygon.

❖ Regular Polygon:
A polygon is called a regular polygon if all its sides have equal length and all the angles have equal measure.

❖ REMARKS:
(1) The sum of the interior angles of a convex polygon of n sides is (2n – 4) right angles or (2n – 4) 90º.
(2) The sum of the exterior angles of a convex polygon is 4 right angles or 360º.
(2 n − 4)  90 
(3) Each interior angle of a n–sided regular polygon is
n
 360  
(4) Each exterior angle of a regular polygon of n sides =  .
 n 
n(n − 3)
(5) If a polygon has n sides, then the number of diagonals of the polygon = .
2

PW OLYMPIAD WALLAH 10
IOQM

Solved Examples

Example : If the sum of interior angles of a polygon is 1620º, find its number of sides.
Solution : We know that sum of all interior angles of a n sided polygon = (2n – 4) right angles
= [(2n – 4)  90°]
but given sum of interior angles = 1620º
 (2n – 4)  900 = 1620º
1620 
 2n – 4 =
90 
 2n – 4 = 18
 2n = 22
 n = 11.

CLOCK TEST

❖ Minute hand and hour hand coincides once in every hour. They coincide 11 times in 12 hours and 22
times in 24 hours.
❖ They coincide only one time between 11 to 1 O' clock. at 12 O' clock.
❖ Minute hand and hour hand are opposite once in every hour. They do it 11 times in 12 hours and 22
times in 24 hours.
❖ They opposite only one time between 5 to 7 O' clock. at 6 O' clock.
❖ Both hands (minute and hour) are perpendicular twice in every hour. 22 times in 12 hours and 44
times in 24 hours.
❖ In one minute, hour hand moves 1/2º and minute hand moves 6º. In one hour, hour hand moves 30º and
minute hand moves 360º.
❖ In an hour, minute hand moves 55 minutes ahead of hour hand.

Solved Examples

Example : At what time between 3 O' Clock and 4 O' Clock will the two hands coincide?
Solution : At 3 O' clock the distance between the two hands is 15 minutes when they coincide with each
other the distance between the two hands will be 0 min.
So, the time taken (15 + 0) = 15 minutes.
 Minute hand is 55 min. ahead of hour hand in 60 min.

60
 Minute hand is 1 min. ahead of hour hand in min.
55
60  15 180 4
 Minute hand is 15 min. ahead of hour hand in = = 16 min.
55 11 11
4
Hence the right time is 16 minute past 3.
11

PW OLYMPIAD WALLAH 11
IOQM
Example : At what time between 2 O' clock and 3 O' clock will the two hands be opposite?
Solution : At 2 O' clock the distance between the two hands is 10 minutes. When they are at 30 minutes
distance, they are opposite to each other. The time taken (30 + 10) = 40 min.
Minute hand is 55 min. ahead of hour hand in 60 min.

60
 Minute hand is 1 min. ahead of hour hand in min.
55
60  40 480 7
 Minute hand is 40 minutes ahead of hour hand in = = 43 min.
55 11 11
7
Hence, the right time is 43 min. past 2.
11

Example : At what time between 4 O' clock and 5 O' clock will the hands are perpendicular?
Solution : At 4 O' clock the distance between the two hands is 20 min. When they are at 15 minutes
distance, they are perpendicular to each other.
Case–I When the time taken (20 – 15) = 5 min.
Minute hand is 55 min. ahead of hour hand in 60 min.

60  5 60 5
 Minute hand is 5 min. ahead of hour hand in = =5 min.
55 11 11
5
Hence, the right time is 5 min. past 4.
11
Case–II When the time taken (20 + 15) = 35 min.
Minute hand is 55 min. ahead of hour hand in 60 min.

60  35 420 2
 Minute hand is 35 min. ahead of hour hand in = = 38 min.
55 11 11
2
Hence, the right time is 38 min. past 4.
11

MIRROR IMAGE OF CLOCK

❖ If the time is between 1 O' clock to 11 O' clock, then to find the mirror image, time is subtracted from
11 : 60.
❖ If the time is between 11 O' clock to 1 O' clock, then to find the mirror image, time is subtracted from
23 : 60.

Solved Examples

Example : The time in the clock is 4 : 46, what is the mirror image ?
Solution : (11 : 60) – (4 : 46) = 7 : 14.

Example : The time in the clock is 12 : 35, then find its mirror image.
Solution : (23 : 60) – (12 : 35) = 11 : 25.

PW OLYMPIAD WALLAH 12
IOQM
ANGLE BETWEEN TWO HANDS
❖ Angle are of two types:
Positive angle: It is obtained by moving from hour hand to minute hand moving in clockwise direction.
Negative angle: It is obtained by moving from minute hand to hour hand.
❖ Both types of angles are 360º in total. If one angle is known, other can be obtained by subtracting from
360º.

Solved Examples

Example : At 4 : 30, what is the angle formed between hour hand and minute hand ?
Solution. At 4 O' clock angle between hour and min. hand is of 120º.
 In 30 min. minute hand make an angle of 180º.

So, the resultant angle is 180º – 120º = 60º.


But in 30 min. hour hand will also cover an angle of 15º.
Hence, the final angle between both hands is 60º – 15º = 45º.

TRIANGLE
A plane figure bounded by three lines in a plane is called a triangle. Every triangle have three sides and three
angles. If ABC is any triangle then AB, BC & CA are three sides and A, B and C are three angles.

❖ Types of triangles:
(a) On the basis of sides we have three types of triangle.
1. Scalene triangle – A triangle in which no two sides are equal is called a scalene triangle.
2. Isosceles triangle – A triangle having two sides equal is called an isosceles triangle.
3. Equilateral triangle – A triangle in which all sides are equal is called an equilateral triangle.
(b) On the basis of angles we have three types:
1. Right triangle – A triangle in which any one angle is right angle is called right triangle.
2. Acute triangle – A triangle in which all angles are acute is called an acute triangle.
3. Obtuse triangle – A triangle in which any one angle is obtuse is called an obtuse triangle.

❖ SOME IMPORTANT THEOREMS:


Theorem: The sum of interior angles of a triangle is 180º.
Theorem: If the bisectors of angles ABC and ACB of a triangle ABC meet at a point O, then
1
BOC = 90º + A.
2

PW OLYMPIAD WALLAH 13
IOQM
Exterior Angle of a Triangle:
If the side of the triangle is produced, the exterior angle so formed is equal to the sum of two interior
opposite angles.
Corollary: An exterior angle of a triangle is greater than either of the interior opposite angles.
Theorem: The sides AB and AC of a ABC are produced to P and Q respectively. If the bisectors of PBC
1
and QCB intersect at O, then BOC = 90º – A.
2

Solved Examples

Example : In figure, TQ and TR are the bisectors of Q and R respectively. If QPR = 80º and PRT
= 30º, determine TQR and QTR.
Solution : Since the bisectors of Q and R meet at T.
1
 QTR = 90º + QPR
2

1
 QTR = 90º + (80º)
2
 QTR = 90º + 40º = 130º
In QTR, we have
TQR + QTR + TRQ = 180º
 TQR + 130º + 30º = 180º [ TRQ = PRT = 30º]

 TQR = 20º
Thus, TQR = 20º and QTR = 130º.

CONGRUENT FIGURES
The figures are called congruent if they have same shape and same size. In other words, two figures are
called congruent if they are having equal length, width and height.

Fig.(i) Fig.(ii)

In the above figures {fig.(i) and fig.(ii)} both are equal in length, width and height, so these are
congruent figures.

PW OLYMPIAD WALLAH 14
IOQM
(a) Congruent Triangles:
Two triangles are congruent if and only if one of them can be made to superimposed on the other, so as
to cover it exactly.

If two triangles ABC and DEF are congruent then A = D, B = E, C = F and AB = DE,
BC = EF, AC = DF.
If two ABC & DEF are congruent then we write  ABC   DEF, we can not write as  ABC  
DFE or  ABC   EDF.
Hence, we can say that “ two triangles are congruent if and only if there exists a one–one
correspondence between their vertices such that the corresponding sides and the corresponding angles
of the two triangles are equal.

(b) Sufficient Conditions for Congruence of two Triangles:


(i) SAS Congruence Criterion:

Two triangles are congruent if two sides and the included angle of one are equal to the corresponding
sides and the included angle of the other triangle.
(ii) ASA Congruence Criterion:

Two triangles are congruent if two angles and the included side of one triangle are equal to the
corresponding two angles and the included side of the other triangle.

(iii) AAS Congruence Criterion:

If any two angles and a non included side of one triangle are equal to the corresponding angles and side
of another triangle, then the two triangles are congruent.

PW OLYMPIAD WALLAH 15
IOQM
(iv) SSS Congruence Criterion:

Two triangles are congruent if the three sides of one triangle are equal to the corresponding three sides
of the other triangle.

(v) RHS Congruence Criterion :

Two right triangles are congruent if the hypotenuse and one side of one triangle are respectively equal
to the hypotenuse and one side of the other triangle.

❖ Mind it:
If two triangles are congruent then their corresponding sides and angles are also congruent by CPCT
(corresponding parts of congruent triangles are also congruent).

Theorem: If the bisector of the vertical angle bisects the base of the triangle, then the triangle is
isosceles.

Solved Examples

Example : In figure, line m is the bisector of an angle A and O is any point on m. OQ and OP are
perpendiculars from O to the arms of A. Prove that:

(i) AOQ  AOP (ii) OQ = OP.


Solution : (i) In AOQ and AOP,
OAQ = OAP [ line m is the bisector of A]

AQO = APO = 90º [Given]


and AO = AO [Common]
By AAS congruency
AOQ  AOP
(ii) OQ = OP [C.P.C.T.]

PW OLYMPIAD WALLAH 16
IOQM
1
Example : If D is the mid–point of the hypotenuse AC of a right triangle ABC, prove that BD = AC.
2
Solution : Given: ABC is a right triangle such that B = 90° and D is mid point of AC.
1
To prove: BD = AC.
2
Construction: Produce BD to E such that BD = DE and join EC.
Proof:

In ADB and CDE


AD = DC [Given]
BD = DE [By construction]
And, ADB = CDE [Vertically opposite angles]
 By SAS criterion of congruence we have
 ADB   CDE
 EC = AB and CED = ABD ... (i) [By CPCT]]
But CED & ABD are alternate interior angles
 CE || AB
 ABC + ECB = 180° [Consecutive interior angles]
90 + ECB = 180°  ECB = 90°.
Now, In ABC & ECB we have
AB = EC [By (i)]
BC = BC [Common]
And, ABC = ECB = 90°
 By SAS criterion of congruence
 ABC   ECB
 AC = EB [By CPCT]
1 1
 AC = EB
2 2
1
 BD = AC. Hence Proved.
2

SOME INEQUALITY RELATIONS IN A TRIANGLE


In triangle ABC, AB = c, BC = a & CA = b then,

PW OLYMPIAD WALLAH 17
IOQM
(i) The sum of any two sides of a triangle is greater than its third side. i.e. in +ABC,
(A) a + b > c (B) b + c > a (C) a + c > b

(ii) If two sides of a triangle are unequal, then the longer side has greater angle opposite to it
i.e. in ABC, if AB > AC then C > B.

(iii) Of all the line segments that can be drawn to a given line, from a point, not lying on it, the
perpendicular line segment is the shortest. i.e. in PMN,

(A) PM < PN.


(iv) The difference between any two sides of a triangle is less than its third side. i.e. in ABC,
(A) a – b < c (B) b – c < a (C) a – c < b

(v) In a right angle triangle the sum of squares of two smaller sides is equal to the square of its third
side.
i.e. in ABC, a2 + b2 = c2.

(vi) If sum of squares of two smaller sides is greater than the square of its third side then that triangle is
acute angled triangle.
i.e in ABC, a2 + b2 > c2.

(vii) If sum of squares of two smaller sides is lesser than the square of its third side then that triangle is
obtuse angled triangle.
i.e in ABC, a2 + b2 < c2.

Solved Examples

Example : From which given triplet we can make the sides a triangle.
(i) {15, 7, 8}
(ii) {3.5, 4.5, 5.5}
Solution : As we know sum of two sides is always greater then third side.
(i) we can not make the triangle because here the sum of two side is equal to third side
i.e 7 + 8 = 15
(ii) We can make the triangle because sum of two sides is always greater then third side
3.5 + 4.5 >5.5
3.5 + 5.5 >4.5
4.5 + 5.5 >3.5

PW OLYMPIAD WALLAH 18
IOQM
Example : Prove that any two sides of the triangle are together greater than twice the median drawn to the
third side.
Solution : Given: ABC and AD is the median.
To prove: AB + AC > 2AD
Construction: Produce AD to E such that AD = DE. Join EC.
Proof: In ADB and CDE
AD = DE [By construction]
BD = DC [AD is the median]
 ADB =  CDE [Vertically opposite angles]
ADB  CDE [By SAS congruency]
So, by CPCT
AB = EC
In AEC
AC + EC > 2AD. [Sum of two sides of a triangle is always greater than the third side]
So, AC + AB > 2AD. [As EC = AB].

QUADRILATERAL
A quadrilateral is a four sided closed figure.

Let A, B, C and D be four points in a plane such that:


(i) No three of them are collinear.
(ii) The line segments AB, BC, CD and DA do not intersect except at their end points, then figure
obtained by joining A, B, C & D is called a quadrilateral.
Convex and Concave Quadrilaterals:
(i) A quadrilateral in which the measure of each interior angle is less than 180° is called a convex quadrilateral.
In figure, PQRS is convex quadrilateral.

(ii) A quadrilateral in which the measure of one of the interior angles is more than 180° is called a
concave quadrilateral. In figure, ABCD is concave quadrilateral.

PW OLYMPIAD WALLAH 19
IOQM
Special Quadrilaterals:
(i) Parallelogram: A parallelogram is a quadrilateral in which both pairs of opposite sides are
parallel.
In figure, AB || DC, AD || BC therefore, ABCD is a parallelogram.

Properties:
(a) A diagonal of a parallelogram divides it into two congruent triangles.
(b) In a parallelogram, opposite sides are equal.
(c) The opposite angles of a parallelogram are equal.
(d) The diagonals of a parallelogram bisect each other.

(ii) Rectangle: A rectangle is a parallelogram, in which each of its angle is a right angle. If ABCD is
a rectangle then A = B = C = D = 90°, AB = CD, BC = AD and diagonals AC = BD.

(iii) Rhombus: A rhombus is a parallelogram in which all its sides are equal in length. If ABCD is a
rhombus then, AB = BC = CD = DA.

The diagonals of a rhombus are perpendicular to each other.

(iv) Square: A square is a parallelogram having all sides equal and each angle equal to right angle. If
ABCD is a square then AB = BC = CD = DA, diagonal AC = BD and  A =  B =  C =  D
= 90°.

The diagonals of a square are perpendicular to each other.


PW OLYMPIAD WALLAH 20
IOQM
(v) Trapezium: A trapezium is a quadrilateral with only one pair of opposite sides parallel. In figure,
ABCD is a trapezium with AB || DC.

(vi) Kite: A kite is a quadrilateral in which two pairs of adjacent sides are equal. If ABCD is a kite then
AB = AD and BC = CD.

(vii) Isosceles trapezium: A trapezium is said to be an isosceles trapezium, if its non–parallel sides
are equal. Thus a quadrilateral ABCD is an isosceles trapezium, if AB || DC and AD = BC.

In isosceles trapezium A = B and C = D.

PROPERTIES

Theorem 1 : The sum of the four angles of a quadrilateral is 360°.


Theorem 2 : A diagonal of a parallelogram divides it into two congruent triangles.
Theorem 3 : In a parallelogram, opposite sides are equal.
Theorem 4 : The opposite angles of a parallelogram are equal.
Theorem 5 : The diagonals of a parallelogram bisect each other.
Theorem 6 : Each of the four angles of a rectangle is a right angle.
Theorem 7 : Each of the four sides of a rhombus is of the same length.
Theorem 8 : Each of the angles of a square is a right angle and each of the four sides is of the
same length.
Theorem 9 : The diagonals of a rectangle are of equal length.
Theorem 10 : The diagonals of a rhombus are perpendicular to each other.
Theorem 11 : The diagonals of a square are equal and perpendicular to each other.
Theorem 12 : Parallelogram and Triangles on the same base (or equal bases) and between the same
parallels, then area of parallelogram is twice the area of triangle.
Theorem 13 : Two triangles on the same base (or equal bases) and between the same parallels are
equal in area.
Theorem 14 : Median of a triangle divides it into two triangles of equal area.
Theorem 15 : Parallelograms on the same base and between the same parallels are equal in area.

PW OLYMPIAD WALLAH 21
IOQM
A quadrilateral become a parallelogram when:
(i) Opposite angles are equal.
(ii) Both the pair of opposite sides are equal
(iii) A pair of opposite side is equal as well as parallel
(iv) Diagonals of quadrilateral bisect each other.

❖ REMARK:
(i) Square, rectangle and rhombus are all parallelograms.
(ii) Kite and trapezium are not parallelograms.
(iii) A square is a rectangle.
(iv) A square is a rhombus.
(v) A parallelogram is a trapezium.

Solved Examples

Example : The diagonals of a parallelogram ABCD intersect at O. A line through O intersects AB at X


and DC at Y. Prove that OX = OY.
Solution : Since AB || CD.

In OAX and OCY


1 = 2 [Alternate angles]
OA = OC and, 3 = 4 [Vertically opposite angles]
So, by ASA criterion of congruence,
OAX  OCY
 OX = OY.

Example : The angle bisectors of a parallelogram form a rectangle.


Solution : A parallelogram ABCD in which bisectors of angles A, B, C, D intersect at P, Q, R, S to form a
quadrilateral PQRS.

Since, ABCD is a parallelogram, so AD || BC and transversal AB intersects them at A and B


respectively.
1 1
A + B = 180º  A + B = 90º
2 2
 BAS + ABS = 90º ....(i)
[AS and BS are bisectors of A and B respectively]

PW OLYMPIAD WALLAH 22
IOQM
BAS + ABS + ASB = 180º  90º + ASB = 180º
 ASB = 90º  RSP = 90º
[ASB and RSP are vertically opposite angles]
Similarly, SRQ = 90º, RQP = 90º and SPQ = 90º.
Hence, PQRS is rectangle.

Example : ABCD is a trapezium in which AB || CD and AD = BC. Show that:


(i) A = B (ii) C = D
(iii)  ABC   BAD (iv) diagonal AC = diagonal BD.
Solution :

(i) Extend AB and draw a line through C parallel to DA intersecting AB produced at E.


Since, AD || CE and transversal AE cuts them at A and E respectively.
 A + E = 180º  180º – E = A
Since, AB || CD and AD || CE
 AECD is a parallelogram.  AD = CE  BC = CE
Thus, in BCE
BC = CE  CBE = CEB  180º – B = E
 180º – E = B  A = B.
(ii) Consecutive interior angles on the same side of a transversal are supplementary.
 A + D = 180º and E + C = 180º  A + D = E + C
 B + D = E + C  D = C [B = E]
(iii) In ABC and BAD
AB = BA
B = A and BC = AD
So, by SAS congruence criterion
ABC  BAD
(iv) Since, ABC  BAD
AC = BD
Hence, diagonal AC = diagonal BD.

Example : In a parallelogram ABCD, AB = 8 cm. The altitudes corresponding to sides AB and AD are
respectively 4 cm and 5 cm. Find AD.
Solution : Area of a ||gm = Base × corresponding altitude

 Area of parallelogram ABCD = AD × BN = AB × DM


8 4
 AD × 5 = 8 × 4  AD = = 6.4 cm.
5
PW OLYMPIAD WALLAH 23
IOQM
Example : ABCD is a quadrilateral. A line through D, parallel to AC, meets BC produced in P as shown in
figure. Prove that ar (ABP) = ar(quad. ABCD).

Solution : Since s ACP and ACD are on the base AC and between the same parallels AC and DP.
 ar(ACP) = ar(ACD)  ar(ACP) + ar(ABC) = ar(ACD) + ar(ABC)
 ar(ABP) = ar(quad. ABCD).

Example : If the medians of a  ABC intersect at G,


1
show that ar( AGB) = ar( AGC) = ar( BGC) = ar( ABC).
3
Solution : Given: A  ABC and its medians AD, BE and CF intersect at G.
1
To prove: ar( AGB) = ar( AGC) = ar( BGC) = ar( ABC).
3
Proof: A median of a triangle divides it into two triangles of equal area.
In  ABC, AD is the median.
 ar( ABD) = ar(ACD) ......(i)

In GBC, GD is the median.


 ar(GBD) = ar(GCD) ...(ii)
Subtract equation (ii) from (i), we get
ar(ABD) – ar(GBD) = ar( ACD) – ar(GCD)
 ar(AGB) = ar(AGC) ......(iii)
Similarly, ar( AGB) = ar( BGC) ......(iv)
From (iii) & (iv)
ar( AGB) = ar( AGC) = ar(BGC)
But, ar(ABC) = ar( AGB) + ar( AGC) + ar( BGC) = 3 ar ( AGB)
1
 ar( AGB) = ar( ABC).
3
1
Hence, ar( AGB) = ar( AGC) = ar(BGC) = ar( ABC).
3

PW OLYMPIAD WALLAH 24
IOQM
MID POINT THEOREM

In a triangle, the line segment joining the mid–points of any two sides is parallel to the third side and is
half of it.

Given: A triangle ABC in which P is the mid–point of side AB and Q is the mid–point of side AC.
To Prove: PQ is parallel to BC and is half of it
1
i.e., PQ || BC and PQ = BC.
2
Construction: Produce PQ upto point R such that PQ = QR. Join R and C.
Proof: In APQ and CRQ
PQ = QR [By construction]
AQ = QC [Given]
And, AQP = CQR [Vertically opposite angles]
So, APQ  CRQ [By SAS]
 AP = CR [By cpct]
And, APQ = CRQ [By cpct]
But, APQ and CRQ are alternate angles and whenever the alternate angles are equal, the lines are
parallel.
 AP || CR  AB || CR  BP || CR
AP = BP [Given, P is mid–point of AB]
 CR = BP [As, AP = CR]
Now, BP = CR and BP || CR  BCRP is a parallelogram.
[When any pair of opposite sides are equal and parallel, the quadrilateral is a parallelogram]
BCRP is a parallelogram and opposite sides of a parallelogram are equal and parallel.
 PR = BC and PR || BC
1 1
Since, PQ = QR  PQ = PR = BC [As, PR = BC]
2 2
Also, PQ || BC [As, PR || BC]
1
 PQ || BC and PQ = BC Hence Proved.
2

Converse of the Mid–Point Theorem: The line drawn through the mid–point of one side of a triangle
parallel to the another side; bisects the third side.

PW OLYMPIAD WALLAH 25
IOQM
Given: A triangle ABC in which P is the mid–point of side AB and PQ is parallel to BC.
To prove: PQ bisects the third side AC i.e., AQ = QC.
Construction: Through C, draw CR parallel to BA, which meets PQ produced at point R.
Proof: Since, PQ || BC i.e., PR || BC [Given]
and CR || BA i.e., CR || BP. [By construction]
 Opposite sides of quadrilateral PBCR are parallel.
 PBCR is a parallelogram
 BP = CR
Also, BP = AP [As, P is mid–point of AB]
 CR = AP

 AB || CR and AC is transversal, PAQ = RCQ [Alternate angles]

 AB || CR and PR is transversal, APQ = CRQ [Alternate angles]


In  APQ and CRQ
CR = AP, PAQ = RCQ and APQ = CRQ
 APQ  CRQ [By ASA]
 AQ = QC Hence Proved.

Solved Examples

Example : In the adjoining figure, D, E, F are the midpoints of the sides BC, CA and AB of ABC. If BE
1
and DF intersect at X while CF and DE intersect at Y, prove that XY = BC.
4
Solution : In ABC, F and E are the midpoints of AB and AC respectively.

1
 FE || BC and FE = BC = BD.
2

 FE || BD and FE = BD.
So, BDEF is a parallelogram whose diagonals BE and DF intersect each other at X.
 X is the midpoint of DF.
Similarly, Y is the midpoint of DE.
Thus, in DEF, X and Y are the midpoints of DF and DE respectively.
1 1 1 1
So, XY || FE and XY = FE = × BC = BC.
2 2 2 4

PW OLYMPIAD WALLAH 26
IOQM
Example : In the adjoining figure, ABCD is a trapezium in which AB || DC and AD = BC. If P, Q, R, S be
respectively the midpoints of BA, BD and CD, CA then show that PQRS is a rhombus.
Solution : In BDC, Q and R are the midpoints of BD and CD respectively.
1
 QR || BC and QR = BC.
2
1
Similarly, PS || BC and PS = BC.
2
1
 PS || QR and PS = QR [Each equal to BC.]
2
 PQRS is a parallelogram.
In ACD, S and R are the midpoints of AC and CD respectively.
1 1
 SR || AD and SR = AD = BC [ AD = BC]
2 2
 PS = QR = SR = PQ.
Hence, PQRS is a rhombus.

Example : Prove that the line segment joining the midpoints of the diagonals of a trapezium is parallel to
the parallel sides and equal to half of their difference.
Solution : Let ABCD be a trapezium in which AB || DC, and let M and N be the midpoints of the
diagonals AC and BD respectively.

Join CN and produce it to meet AB at E.


In CDN and EBN, we have
DN = BN  N is midpoint of BD]

DCN = BEN [Alt. Int. ]


CDN = EBN [Alt. Int. ]
 CDN  EBN [AAS–criteria]
 DC = EB and CN = NE [c.p.c.t]
Thus; in CAE, the points M and N are the midpoints of AC and CE respectively.
1
 MN || AE and MN = AE  MN || AB || DC
2
1 1 1
and MN = AE = (AB – EB) = (AB – DC) [ EB = DC].
2 2 2

SIMILAR FIGURES
Two geometric figures having the same shape and size are known as congruent figures. Geometric
figures having the same shape but different sizes are known as similar figures.

PW OLYMPIAD WALLAH 27
IOQM
SIMILAR TRIANGLES
Two triangles ABC and DEF are said to be similar if their
(i) Corresponding angles are equal.

i.e. A = D, B = E, C = F

(ii) Corresponding sides are proportional. i.e.


AB BC AC
= = .
DE EF DF
(a) Characteristic Properties of Similar Triangles:
(i) (AAA Similarity) If two triangles are equiangular, then they are similar.
(ii) (SSS Similarity) If the corresponding sides of two triangles are proportional, then they are similar.
(iii) (SAS Similarity) If in two triangles, two pairs of corresponding sides are proportional and the
included angles are equal then the two triangles are similar.

(b) Results Based Upon Characteristic Properties of Similar Triangles:


(i) If two triangles are equiangular, then the ratio of the corresponding sides is the same as the ratio of
the corresponding medians.
(ii) If two triangles are equiangular, then the ratio of the corresponding sides is same as the ratio of the
corresponding angle bisector segments.
(iii) If two triangles are equiangular then the ratio of the corresponding sides is same as the ratio of the
corresponding altitudes.
(iv) If one angle of a triangle is equal to one angle of another triangle and the bisectors of these equal
angles divide the opposite side in the same ratio, then the triangles are similar.
(v) If two sides and a median bisecting one of these sides of a triangle are respectively proportional to
the two sides and the corresponding median of another triangle, then the triangles are similar.
(vi) If two sides and a median bisecting the third side of a triangle are respectively proportional to the
corresponding sides and the median of another triangle, then two triangles are similar.
❖ Mind it:
1 1 1
If the ratio of sides of triangle is a : b : c, then ratio of their altitudes is : : .
a b c
THALES THEOREM (BASIC PROPORTIONALITY THEOREM)
Statement: If a line is drawn parallel to one side of a triangle to intersect the other two sides in distinct
points, then the other two sides are divided in the same ratio.
Given: A triangle ABC in which a line parallel to side BC intersects other two sides AB and AC at D
and E respectively.

PW OLYMPIAD WALLAH 28
IOQM
AD AE
To Prove: = .
BD EC
Construction: Join BE and CD and draw DM ⊥ AC and EN ⊥ AB.
1 1
Proof: Area of  ADE = base × height = AD × EN.
2 2

Area of  ADE is denoted as ar(ADE)


1
So, ar(ADE) = AD × EN
2
1
And ar(BDE) = DB × EN,
2
1
AD  EN
ar(ADE)
2 AD
Therefore, = = ... (i)
ar(BDE) 1 DB
DB  EN
2

1 1
Similarly, ar(ADE) = AE × DM and ar(DEC) = EC × DM.
2 2
1
AE  DM
ar(ADE) AE
And = 2 = ... (ii)
ar(DEC) 1 EC
EC  DM
2

Note that  BDE and  DEC are on the same base DE and between the two parallel lines BC and DE.
So, ar(BDE) = ar(DEC) ... (iii)
Therefore, from (i), (ii) and (iii), we have:
AD AE
= Hence Proved.
DB EC

COROLLARY

If in a ABC, a line DE || BC, intersects AB in D and AC in E, then

DB EC AB AC
(i) = (ii) =
AD AE AD AE
AD AE AB AE
(iii) = (iv) =
AB AC DB EC
DB EC
(v) =
AB AC

PW OLYMPIAD WALLAH 29
IOQM
(a) Converse of Basic Proportionality Theorem:
If a line divides any two sides of a triangle in the same ratio, then the line must be parallel to the third
side.
(b) Some Important Results and Theorems:
(i) The internal bisector of an angle of a triangle divides the opposite side internally in the ratio of the
sides containing the angle.
(ii) In a triangle ABC, if D is a point on BC such that D divides BC in the ratio AB : AC, then AD is
the bisector of A.
(iii) The external bisector of an angle of a triangle divides the opposite sides externally in the ratio of
the sides containing the angle.
(iv) The line drawn from the mid–point of one side of a triangle parallel to another side bisects the third
side.
(v) The line joining the mid–points of two sides of a triangle is parallel to the third side.
(vi) The diagonals of a trapezium divide each other proportionally.
(vii) If the diagonals of a quadrilateral divide each other proportionally, then it is a trapezium.
(viii) Any line parallel to the parallel sides of a trapezium divides the non–parallel sides proportionally.
(ix) If three or more parallel lines are intersected by two transversals, then the intercepts made by them
on the transversals are proportional.

Solved Examples

Example : In a ABC, D and E are points on the sides AB and AC respectively such that DE || BC.
If AD = 4x – 3, AE = 8x – 7, BD = 3x – 1 and CE = 5x – 3, find the value of x.
Solution : In ABC, we have
DE || BC
AD AE
 = [By Basic Proportionality Theorem]
DB EC
4x − 3 8x − 7
 =
3x − 1 5x − 3
 20x2 – 15x – 12x + 9 = 24x2 – 21x – 8x + 7
 20x2 – 27x + 9 = 24x2 – 29x + 7
 4x2 – 2x – 2 = 0
 2x2 – x – 1 = 0
 (2x + 1) (x – 1) = 0
1
 x = 1 or x = –
2
So, the required value of x is 1.
1
[x = – is neglected as length can not be negative].
2

PW OLYMPIAD WALLAH 30
IOQM
Example : D and E are respectively the points on the sides AB and AC of a ABC such that AB = 12 cm,
AD = 8 cm, AE = 12 cm and AC = 18 cm, show that DE || BC.
Solution : We have,
AB = 12 cm, AC = 18 cm, AD = 8 cm and AE = 12 cm.
 BD = AB – AD = (12 – 8) cm = 4 cm
CE = AC – AE = (18 – 12) cm = 6 cm
AD 8 2
Now, = =
BD 4 1
AE 12 2 AD AE
And, = =  =
CE 6 1 BD CE
Thus, DE divides sides AB and AC of ABC in the same ratio. Therefore, by the converse of
basic proportionality theorem, we have DE || BC.

Example : In a trapezium ABCD, AB || DC and DC = 2AB. EF drawn parallel to AB cuts AD in F and BC


BE 3
in E such that = . Diagonal DB intersects EF at G. Prove that 7FE = 10AB.
EC 4
Solution : In DFG and DAB,
1 = 2 [Corresponding s AB || FG]

FDG = ADB [Common]


 DFG ~ DAB [By AA rule of similarity]
DF FG
 = ......(i)
DA AB
FDG = ADB [Common]
 DFG ~ DAB [By AA rule of similarity]
DF FG
 = .......(i)
DA AB
Again in trapezium ABCD
EF || AB || DC
AF BE
 =
DF EC
AF 3  BE 3 
 =  EC = 4 (given) 
DF 4  
AF 3
 +1= +1
DF 4
AF + DF 7
 =
DF 4
AD 7 DF 4
 =  = .....(ii)
DF 4 AD 7
From (i) and (ii), we get
FG 4 4
= i.e., FG = AB ......(iii)
AB 7 7
In BEG and BCD, we have
BEG = BCD [Corresponding angle  EG || CD]
PW OLYMPIAD WALLAH 31
IOQM
GBE = DBC [Common]
 BEG ~ BCD [By AA rule of similarity]
BE EG
 =
BC CD
3 EG  BE 3 EC 4 EC + BE 4 + 3 BC 7 
 =  EG = 7 i.e., BE = 3  =  =
7 CD  BE 3 BE 3 
3 3
 EG = CD = (2 AB) [ CD = 2AB (given)]
7 7
6
 EG = AB .....(iv)
7
Adding (iii) and (iv), we get
4 6 10
FG + EG = AB + AB = AB
7 7 7
10
 EF = AB i.e., 7EF = 10AB. Hence proved.
7

Area (  ABD) AB
Example : In ABC, if AD is the bisector of A, prove that = .
Area (  ACD) AC
Solution : In ABC, AD is the bisector of A.

AB BD
 = .....(i) [By internal bisector theorem]
AC DC
From A draw AL ⊥ BC
1
BD.AL
Area (  ABD) 2 BD AB
 = = = [From (i)]
Area (  ACD) 1
DC.AL
DC AC
2
Hence Proved.

Example : In triangle ABC, BAC = 90º, AD is its bisector. If DE ⊥ AC, prove that DE × (AB + AC) =
AB × AC.
Solution : It is given that AD is the bisector of A of  ABC.
AB BD
 =
AC DC
AB BD
 +1= +1 [Adding 1 on both sides]
AC DC
AB + AC BD + DC
 =
AC DC
AB + AC BC
 = .....(i)
AC DC

PW OLYMPIAD WALLAH 32
IOQM
In 's CDE and CBA, we have
DCE = BCA [Common]
DEC = BAC [Each equal to 90º]
So, by AA–criterion of similarity
 CDE ~  CBA
CD DE
 =
CB BA
AB BC
 = .....(ii)
DE DC
From (i) and (ii), we have
AB + AC AB
 =
AC DE
 DE × (AB + AC) = AB × AC.

AREAS OF SIMILAR TRIANGLES


Theorem: The ratio of the areas of two similar triangles is equal to the square of the ratio of their
corresponding sides.
Given: Two triangles ABC and PQR such that  ABC ~  PQR [Shown in the figure]

2 2 2
ar(  ABC)  AB   BC   CA 
To prove: =  =  =  .
ar(  PQR)  PQ   QR   RP 
Construction: Draw altitudes AM and PN of the triangle ABC and PQR.
1
Proof: ar(ABC) = BC × AM
2
1
And ar(PQR) = QR × PN
2
1
BC  AM
ar(ABC) BC  AM
So, = 2 = .....(i)
ar(PQR) 1 QR  PN
QR  PN
2
Now, in  ABM and  PQN,
And B = Q [As  ABC   PQR]
M = N [ 90º each ]
So,  ABM   PQN [AA similarity criterion]
AM AB
Therefore, = .....(ii)
PN PQ
Also,  ABC ~  PQR [Given]
AB BC CA
So, = = .....(iii)
PQ QR RP
PW OLYMPIAD WALLAH 33
IOQM
ar(ABC) BC AB
Therefore, =  [From (i) and (ii)]
ar(PQR ) QR PQ

AB AB
=  [From (iii)]
PQ PQ
2
 AB 
= 
 PQ 
Now using (iii), we get
2 2 2
ar(ΔABC)  AB   BC   CA 
=  =  = 
ar(ΔPQR)  PQ   QR   RP 

(a) Properties of Areas of Similar Triangles:


(i) The areas of two similar triangles are in the ratio of the squares of corresponding altitudes.
(ii) The areas of two similar triangles are in the ratio of the squares of the corresponding medians.
(iii) The area of two similar triangles are in the ratio of the squares of the corresponding angle bisector
segments.

Solved Examples

Example : Prove that the area of the equilateral triangle described on the side of a square is half the area of
the equilateral triangle described on its diagonal.
Solution : Given: A square ABCD. Equilateral triangles BCE and ACF have been described on side
BC and diagonal AC respectively.
1
To Prove: Area (BCE) = . Area (ACF)
2
Proof: Since BCE and ACF are equilateral. Therefore, they are equiangular (each angle
being equal to 60º) and hence BCE ~ ACF.

Area(  BCE) BC 2
 =
Area(  AC F) AC 2

Area( BCE) BC 2 1
 = =
Area(  AC F)
( )
2
2BC 2

Area( BCE) 1
 = . Hence Proved.
Area(  AC F) 2

PW OLYMPIAD WALLAH 34
IOQM
PYTHAGORAS THEOREM
Statement: In a right triangle, the square of the hypotenuse is equal to the sum of the squares of the
other two sides.
Given: A right triangle ABC, right angled at B.
To prove: AC2 = AB2 + BC2
Construction: BD ⊥ AC

Proof:  ADB &  ABC


DAB = CAB [Common]
BDA = CBA [90º each]
So,  ADB ~  ABC [By AA similarity]
AD AB
= [Sides are proportional]
AB AC
or, AD . AC = AB2 .....(i)
Similarily  BDC ~  ABC
CD BC
So, =
BC AC
or CD . AC = BC2 .....(ii)
Adding (i) and (ii),
AD . AC + CD . AC = AB2 + BC2
or, AC (AD + CD) = AB2 + BC2
or, AC. AC = AB2 + BC2
or, AC2 = AB2 + BC2 Hence Proved.

(a) Converse of Pythagoras Theorem:


Theorem: In a triangle, if the square of one side is equal to the sum of the squares of the other two
sides, then the angle opposite to the first side is a right angle.

Given: A triangle ABC such that AC2 = AB2 + BC2


Construction: Construct a triangle DEF such that DE = AB, EF = BC and E = 90º.
Proof: In order to prove that B = 90º, it is sufficient to show ABC ~ DEF. For this we proceed as
follows.
Since DEF is a right–angled triangle with right angle at E. Therefore, by Pythagoras theorem, we have

PW OLYMPIAD WALLAH 35
IOQM
DF2 = DE2 + EF2  DF2 = AB2 + BC2 [DE = AB and EF = BC (By construction)]

 DF2 = AC2 [ AB2 + BC2 = AC2 (Given)]

 DF = AC ......(i)
Thus, in  ABC and  DEF, we have
AB = DE, BC = EF [By construction]
And AC = DF [From equation (i)]
  ABC   DEF [By SSS criteria of congruency]
 B = E = 90º
Hence, ABC is a right triangle, right angled at B.

(b) Some Results Deduced From Pythagoras Theorem:


(i)In the given figure ABC is an obtuse triangle, obtuse angled at B.
If AD ⊥ CB, then AC2 = AB2 + BC2 + 2BC. BD

(ii) In the given figure, if  B of ABC is an acute angle and AD ⊥ BC, then
AC2 = AB2 + BC2 –2BC. BD

(iii) In any triangle, the sum of the squares of any two sides is equal to twice the square of half of the
third side together with twice the square of the median which bisects the third side.
(iv) Three times the sum of the squares of the sides of a triangle is equal to four times the sum of the
squares of the medians of the triangle.

Solved Examples

Example : BL and CM are medians of ABC right angled at A. Prove that 4 (BL2 + CM2) = 5 BC2.
Solution : In BAL
 BL2 = AL2 + AB2 .....(i) [Using Pythagoras theorem]
and, In CAM
CM2 = AM2 + AC2 ..... (ii) [Using Pythagoras theorem]

PW OLYMPIAD WALLAH 36
IOQM
Adding (1) and (2) and then multiplying by 4, we get

4(BL2 + CM2) = 4(AL2 + AB2 + AM2 + AC2)


= 4{AL2 + AM2 + (AB2 + AC2)} [  ABC is a right triangle]
2 2 2
= 4(AL + AM + BC ) = 4(ML + BC ) 2 2
[ LAM is a right triangle]
2 2
= 4ML +4 BC
[A line joining mid–points of two sides is parallel to third side and is equal to half of it, ML =BC/2]
= BC2 + 4BC2 = 5BC2. Hence proved.

Example : In the given figure, BC ⊥ AB, AE ⊥ AB and DE ⊥ AC. Prove that DE. BC = AD. AB.
Solution : In ABC and EDA,
We have
 ABC =  ADE [Each equal to 90°]
 ACB =  EAD [Alternate angles]

 By AA Similarity
ABC ~ EDA
BC AD
 =
AB DE
 DE. BC = AD. AB. Hence Proved.

Example : O is any point inside a rectangle ABCD (shown in the figure). Prove that OB2 + OD2 = OA2 + OC2.
Solution : Through O, draw PQ || BC so that P lies on AB and Q lies on DC.
Now, PQ || BC

Therefore,
PQ ⊥ AB and PQ ⊥ DC [ B = 90º and  C = 90º]
So,  BPQ = 90º and  CQP = 90º
Therefore, BPQC and APQD are both rectangles.
PW OLYMPIAD WALLAH 37
IOQM
Now, from  OPB,
OB2 = BP2 + OP2 ... (i)
Similarly, from  ODQ,
OD2 = OQ2 + DQ2 ... (ii)
From  OQC, we have
OC2 = OQ2 + CQ2 ... (iii)
And from  OAP, we have
OA2 = AP2 + OP2 ... (iv)
Adding (i) and (ii)
OB2 + OD2 = BP2 + OP2 + OQ2 + DQ2
= CQ2 + OP2 + OQ2 + AP2 [As BP = CQ and DQ = AP]
= CQ2 + OQ2 + OP2 + AP2 = OC2 + OA2 [From (iii) and (iv)] Hence Proved.

Example : AD, BE and CF be medians of a ABC.


Prove that 2(AD + BE + CF) < 3(AB + BC + CA) < 4(AD + BE + CF).
Solution : In ABC, AD, BE and CF are medians. We know that these are concurrent. Let these meet at G.
Also AG : GD = BG : GE = CG : GF = 2 : 1
2 2 2
So, AG = AD, BG = BE, CG = CF.
3 3 3
Because in triangle, the sum of two sides is greater than the third,
So, in AGB, AG + BG > AB
in BGC, BG + CG > BC
in CGA, CG + AG > CA
Adding these, we get
2(AG + BG + CG) > AB + BC + CA
2 2 2 
or 2  AD + BE + CF  > AB + BC + CA
3 3 3 
4
or (AD + BE + CF) > AB + BC + CA
3
or 4(AD + BE + CF) > 3(AB + BC + CA) .....(i)
Now, in ABD
AB + BD > AD
1
or, AB + BC > AD (Since AD is a median)
2
1 1
Similarly, BC + CA > BE and CA + AB > CF
2 2
3
Adding these, we get (AB + BC + CA) > AD + BE + CF
2
or 3(AB + BC + CA) > 2(AD + BE + CF) .....(ii)
Combining (i) and (ii), we get
2(AD + BE + CF) < 3(AB + BC + CA) < 4(AD + BE + CF).
PW OLYMPIAD WALLAH 38
IOQM
Example : In a ABC, AC > AB. The bisector of A meets BC at E. Prove that CE > BE.
Solution : Given: A ABC, in which AC > AB and AE bisects A
To Prove: CE > BE.
Construction: Take a point F on AC, such that AF = AB. Join BF and EF. Let BF intersect AE at O.
Proof: ABF is an isosceles (Since AB = AF, by construction)
AE bisects A (given)
 AE is the perpendicular bisector of BF, so any point on AE is equidistant from B and F. In
particular,
BE = FE ......(i)
So, in BEF,
EBF = EFB = , say (angles opposite equal sides)
So, BEO = 90° –  = FEO (= , say)
Thus AE also bisects BEF.
Now, because in a triangle, exterior angle is
the sum of interior opposite angles, so
EFC =  +  and ext B =  + 
So, EFC = ext B. But B = 2 + C, so EFC = 2 + C.
Therefore EFC > C
So, CE > FE
Hence CE > BE (using (i)).

ADVANCED THEOREMS:

Test Property Diagram


A–A–A If in two triangles corresponding angles are equal i.e., the two
(similarity) triangles are equiangular, then the triangles are similar.
A = D, B = E & C = F
ABC ~ DEF
S–S–S if the corresponding sides of two triangles are proportional,
(Similarity) then they are similar.
AB BC AC
= =
DE EF DF
ABC ~ DEF
S–A–S If in two triangles, one pair of corresponding sides are
(Similarity) proportional and the included angles are equal then the two
triangles arc similar.
AB AC
= & BAC = EDF
DE DF
ABC ~ DEF

PW OLYMPIAD WALLAH 39
IOQM
Important Definition
Nomenclature Property / Definition Diagram
Altitude The perpendicular drawn from the opposite vertex of a
(or height) side in a triangle called an altitude of the triangle.
 There are three altitudes in a triangle.

AE, CD and BF are the altitudes


Median The line segment joining the mid-point of a side to the
vertex opposite to the side is called a median.
 There are three medians in a triangle.
 A median bisects the area of the triangle i.e,
1
Ar(ABE) = Ar(AEC) = Ar(ABC) etc.
2 AE, CD and BF are the medians
 Point of intersection is called Centroid. (BE = CE, AD = BD, AF = CF)
Angle bisector A line segment which originates from a vertex and
bisects the same angle is called an angle bisector
1
(BAE = CAE = BAC) etc.
2
 Point of intersection of angle bisectors is called
incentre. AE, CD and BF are the angles
bisectors
Perpendicular A line segment which bisects a side perpendicularly
bisector (i.e. at right angle) is called a perpendicular bisector of
a side of triangle.
 All points on the perpendicular bisector of a line are
equidistant from the ends of the line.
 Point of intersection of perpendicular bisectors is
called Circumcentre. DO, EO and FO are the
perpendicular bisectors
Orthocentre The point of intersection of the three altitudes of the
triangle is called as the orthocentre.
BOC = 180° – A
COA = 180° – B
AOB = l80° –C
'O' is the orthocentre

Theorems related to triangles


Theorems Statement/Explanation Diagram
Vertical angle The bisector of the vertical angle of a triangle divides
bisector the base in the ratio of other two sides.
BD AB
=
DC AC

Apollonius In a triangle, the sum of the squares of any two sides of a


theorem triangle is equal to twice the sum of the square of the median
to the third side and square of half the third side.
i.e. AB2 + AC2 = 2(AD2 + BD2)

BD = CD AD is the median

PW OLYMPIAD WALLAH 40
IOQM
Results on area of similar triangles

S.No. Statement Diagram


1. The areas of two similar triangles are proportional to the
squares of their corresponding sides.
If ABC ~ DEF then
Area of ABC B 2 BC 2 AC 2
= = =
Area of DEF DE 2 EF 2 DF 2
2. The areas of two similar triangles are proportional to the
squares of their corresponding altitude.
If ABC ~ DEF, AL ⊥ BC and DM ⊥ EF
Area of ABC L2
then =
Area of DEF DM 2
3. The areas of two similar triangles are proportional to the
squares of their corresponding medians.
If ABC  DEF and AP, DQ are their medians
Area of ABC P 2
then =
Area of DEF DQ 2
4. The areas of two similar triangles are proportional to the
squares of their corresponding angle bisector segments.
If ABC ~ DEF and AX . DY are their
bisectors of A and D respectively
Area of ABC X 2
then =
Area of DEF DY 2
5. If D, E and Fare respectively the mid-points of sides BC, CA
and AB of an equilateral triangle ABC then DEF is also an
equilateral triangle.

Some useful results


S.No. Statement Diagram
1. In a ABC, if the bisectors of B and C meet at O then
BOC=90° + (A)/2

2. In a ABC, if sides AB and AC are produced to D and E


respectively and the bisectors of DBC and ECB intersect
at O, then
BOC = 90° –(A)/2

PW OLYMPIAD WALLAH 41
IOQM
3. In a ABC, if AD is the angle bisector of BAC and AE ⊥
1
BC, DAE = (ABC – ACB)
2

4. In a ABC, if side BC is produced to D and bisectors of


ABC and ACD meet at E, then
1
BEC = BAC
2
5. In an acute angle ABC, AD is a perpendicular dropped on
the opposite side of A then
AC2 = AB2 + BC2 – 2 BD. BC (B < 90°)

6. In a obtuse angle ABC, AD is perpendicular dropped on


BC. BC is produce to D to meet AD, then
AC2 = AB2 + BC2 + 2 BD BC (B > 90°)

7. In a right angle ABC, B=90° and AC is hypotenuse the


perpendicular BD is dropped on hypotenuse AC from right
angle vertex B, then
AB  BC AB 2
(i) BD = (ii) AD =
AC AC

BC 2 1 1 1
(iii) CD = (iv) 2
= 2
+
AC BD AB BC 2

8. In a right angled triangle, the median to the hypotenuse


1 AC
= × hypotenuse, i.e., BM =
2 2

Triangle inequality theorem


The sum of the lengths of two sides of any triangle is greater than the length of the third side.
a + b > c, a + c > b, b + c > a.
1
If P = a + b + c and a  b  c, then < P
2

PW OLYMPIAD WALLAH 42
IOQM
The sine rule
In any triangle ABC (acute or obtuse angled):
a b c
= =
sin A sin B sin C

The cosine rule


In any triangle ABC (acute–or obtuse–angled):
c2 = a2 + b2 – 2ab cos C.

Similarly, b2 = a2 + c2 – 2ac cos B, and a2 = b2 + c2 – 2bc cos A.

HERON’S FORMULA

Heron (or Hero) of Alexandria (c. 10 – 70 AD) was an anciend Greek mathematician and engineer who
was active in his native city of Alexandria, Roman Egypt. He is considered the greatest experimenter of
antiquity and his work is representative of the Hellenistic scientific tradition.
Heron’s works as an inventor truly reveal his genius but he is also accredited as a mathematician who
delivered a lot to the field with his practical approach. From approximations of square roots and
formulating the area of a triangle to his treatise in geometry, Heron’s contributions are wide ranging.
The ‘Metrica’ is a series of three three books, found by R. Schone in Istanbul in 1896, in which Heron
focuses on calculating areas and volumes of bodies such as pyramids, cones, cylinders, prisms etc.
‘Hero’s formula’ was found in this book which stated the area of a triangle with given sides.
If a, b, c denote the lengths of the sides of a triangle ABC. Then,
Area of ABC = s(s − a)(s − b)(s − c)
a+b+c
s= is the semi–perimeter of ABC.
2
(a) Perimeter and Area of a Triangle:
(i) Right–angled triangle
For an right–angled triangle, let b be the base, h be the perpendicular and d be the hypotenuse. Then
Perimeter = b + h + d
1 1
Area = (Base × Height) = h
2 2

Hypotenuse, d = b2 + h 2 [Pythagoras theorem]

PW OLYMPIAD WALLAH 43
IOQM
(ii) Isosceles right–angled triangle
For an isosceles right-angled triangle, let a be the equal sides, then

Hypotenuse = a2 + a2 = 2a

Perimeter = 2a + 2a

1 1 1
Area = (Base) = (a × a) = a 2.
2 2 2
(iii) Equilateral triangle
For an equilateral triangle, let each side be a, and the height of the triangle is h, then
A = B = C = 60°

BAD = CAD = 30°


AB = BC = AC = a(say)
a
BD = DC =
2
2
 a 3a 2
  + h 2
= a 2
 h2
=
2 4

3
 Height (h) = a
2

1 1 3 3
Area = (Base × Height) = ×a× a= a2
2 2 2 4
Perimeter = a + a + a = 3a.

(b) Application of Heron’s formula


With the help of heron’s formula we can find the area of Quadrilateral whose all side and one diagonal
is given to us, Quadrilateral whose all sides and angle between any two adjacent side is right angle,
Trapezium whose all sides are given and we can find the area of Parallelogram and its types.

PW OLYMPIAD WALLAH 44
IOQM

Solved Examples

Example : The perimeter of a triangular filed is 450 m and its sides are in the ratio 13 : 12 : 5. Find the
area of the triangle.
Solution : Let a = 13x, b = 12x and c = 5x
 Perimeter = 450
13x + 12x + 5x = 450
30x = 450
x = 15
So, the sides of the triangle are:
a = 13 × 15 = 195 m, b = 12 × 15 = 180 m and c = 5 × 15 = 75 m
2s = 195 + 180 + 75
2s = 450
s = 225.
Hence, Area = s(s − a)(s − b)(s − c) = 225(225 − 195)(225 − 180)(225 − 75)

= 225(30)(45)(150) = 6750 m2.

3
Example : The perimeter of an isosceles triangle is 42 cm and its base is times each of the equal sides.
2
Find the length of each side of the triangle, area of the triangle and height of the triangle.
Solution : Let the equal sides be a and unequal side be b.
 2a + b = 42
3
Given: b = a
2
3
2a + a = 42
2
7
a = 42
2
A = 12 cm.
3
So, b= (12) = 18 cm.
2
Also, perimeter = 2s = 42 cm
s = 21 cm.
Area of triangle = 21(21 − 12)(21 − 12)(21 − 18) = 21(9)(9)(3) = 22 7 cm2

1
Area of triangle = × base × height
2
1
27 7 = × 18 × height
2

54 7
Height = = 3 7 cm
18

PW OLYMPIAD WALLAH 45
IOQM
Example : A triangle and parallelogram have the same base and the same area. If the sides of the triangle
are 26 cm, 28 cm and 30 cm, and the parallelogram stands on the base 28 cm, find the height of
the parallelogram.
26 + 28 + 30
Solution : s= = 42 cm
2
 Area of the triangle = 42(42 − 36)(42 − 28)(42 − 30) = 42(16)(14)(12) = 336 cm2
Let h be the height of the parallelogram.
It is given that the triangle and the parallelogram have the same base and same area.
 Area of the parallelogram = 336 cm2
336
 Base × height = 336  28 × h = 336  h= = 12 cm.
28

Example : Find the area of trapezium whose parallel sides are 25 cm, 13 cm and other sides are 15 cm and
15 cm.
Solution : Let ABCD be the given trapezium in which AB = 25 cm, CD = 13 cm, BC = 15 cm and AD = 15 cm.

Now, ADCE is a parallelogram is which AD || CE and AE || CD.


 AE = DE = 13 cm and BE = AB – AE = 25 – 13 = 12 cm.
In BCE, we have
15 + 15 + 12
s= = 21
2
 Area of BCE = s(s − a)(s − b)(s − c)

 Area of BCE = 21(21 − 15)(21 − 15)(21 − 12)

 Area of BCE = 21  6  6  9

= 18 21 cm2 ……(i)
Let h be the height of BCE, then
1
Area of BCE = (Base × Height)
2
1
= × 12 × h = 6h …(ii)
2
From (i) and (ii), we have,
6h = 18 21
 h = 3 21 cm
Clearly, the height of trapezium ABCD is same that of BCE.
1
 Area of trapezium = (AB + CD) × h
2
1
 Area of trapezium = (25 + 13) × 3 21 cm2 = 57 21 cm2.
2

PW OLYMPIAD WALLAH 46
IOQM
Example : Find the percentage increase in the area of a triangle if its each side is doubled.
Solution : Let a, b, c be the sides of the given triangle and s be its semi–perimeter.
1
 s= (a + b + c) ….. (i)
2
The sides of the new triangle are 2a, 2b, and 2c.
Let’s be its semi–perimeter.
1
 s’ = (2a + 2b + 2c) = a + b + c = 2s [Using (i)]
2
Let  = Area of given triangle
= s(s − a)(s − b)(s − c) …..(ii)

And, ’s = Area of new triangle


’ = s '(s'− 2 a)(s'− 2b)(s'− 2 c)

= 2s(2s − 2 a)(2s − 2b)(2s − 2 c) [Using (i)]


’ = 4 [Using (ii)]
 Increase in the area of the triangle
= ’ –  = 4 –  = 3
 3 
 % increase in area =   100 % = 300%.
  

AREA OF PARALLELOGRAMS AND TRIANGLES


(a) Polygon region
Polygon region can be expressed as the union of a finite number of triangular regions in a plane such
that if two of these intersect, their intersection is either a point or a line segment. It is the shaded portion
including its sides as shown in the figure.

Area Axioms:
Every polygonal region R has an area, measured in square units and denoted by ar(R).
(i) Congruent area axiom: If R1 and R2 be two regions such that R1  R2 then ar(R1) = ar (R2).
(ii) Area addition axiom: If R1 and R2 are two polygonal regions, whose intersection is a finite number
of points & line segments such that R = R1  R2, then ar (R) = ar (R1) + ar (R2).
(iii) Rectangular area axiom: If AB = a metre and AD = b metre then,
ar (Rectangular region ABCD) = ab sq.m.

(b) Area of a parallelogram


Base and Altitude of a Parallelogram:
(i) Base: Any side of a parallelogram can be called its base.

PW OLYMPIAD WALLAH 47
IOQM
(ii) Altitude: The length of the line segment which is perpendicular to the base from the opposite side
is called the altitude or height of the parallelogram corresponding to the given base.

(i) DL is the altitude of ||gm ABCD, corresponding to the base AB.


(ii) DM is the altitude of ||gm ABCD, corresponding to the base BC.
Theorem: A diagonal of a parallelogram divides it into two triangles of equal area.
Given: A parallelogram ABCD whose one of the diagonals is BD.
To prove: ar ( ABD) = ar ( CDB).
Proof

In  ABD and  CDB.


AB = DC [Opposite sides of a ||gm]
AD = BC [Opposite sides of a ||gm]
BD = BD [Common side]
  ABD   CDB [By SSS congruency]
 ar (ABD) = ar (CDB) Hence Proved.
Theorem: Parallelograms on the same base and between the same parallels are equal in area.

Given: Two ||gms ABCD and ABEF on the same base AB and between the same parallels AB and FC.
To prove: ar(||gm ABCD) = ar(||gm ABEF)
Proof: In ADF and BCE, we have
AD = BC [Opposite sides of a ||gm]
AF = BE [Opposite sides of a ||gm]
DAF = CBE [ AD || BC and AF || BE]

[Angle between AD and AF = Angle between BC and BE]


  ADF   BCE [By SAS congruency]
 ar( ADF) = ar( BCE) ...(i)
 ar(||gm ABCD) = ar( ABED) + ar(  BCE)
= ar(ABED) + ar(  ADF) [Using (i)]
= ar(||gm ABEF).
Hence, ar(||gm ABCD) = ar(||gm ABEF). Hence Proved.
PW OLYMPIAD WALLAH 48
IOQM
Theorem: The area of parallelogram is the product of its base and the corresponding altitude.

Given: A ||gm ABCD in which AB is the base and AL is the corresponding height.
To prove: Area (||gm ABCD) = AB × AL.
Construction: Draw BM ⊥DC, so that rectangle ABML is formed.
Proof: ||gm ABCD and rectangle ABML are on the same base AB and between the same parallel lines
AB and LC.
 ar(||gm ABCD) = ar(rectangle ABML) = AB × AL.
 area of a ||gm = base × height. Hence Proved.

(c) Area of a Triangle


Theorem: Two triangles on the same base (or equal bases) and between the same parallels are
equal in area.
Given: Two triangles ABC and PBC on the same base BC and between the same parallel lines BC and
AP.
To prove: ar(  ABC) = ar(  PBC).
Construction: Through B, draw BD || CA intersecting AP produced in D and through C, draw CQ ||
BP, intersecting PA produced in Q.

Proof: BD || CA [By construction]


And, BC || DA [Given]
 Quadrilateral BCAD is a parallelogram.
Similarly, Quadrilateral BCQP is a parallelogram.
Now, parallelogram BCQP and BCAD are on the same base BC, and between the same parallels.
 ar (||gm BCQP) = ar (||gm BCAD) ....(i)
Diagonals of a parallelogram divides it into two triangles of equal area.
1
 ar (PBC) = ar (||gm BCQP) ...(ii)
2
1
And ar (ABC) = ar (||gm BCAD) ..(iii)
2
Now, ar (||gm BCQP) = ar (||gm BCAD) [From (i)]
1 1
 ar (||gm BCAD) = ar (||gm BCQP)
2 2
Hence, ar (ABC) = ar (PBC) [Using (ii) and (iii)] Hence Proved.

PW OLYMPIAD WALLAH 49
IOQM
Theorem: If a triangle and a parallelogram are on the same base and between the same parallels,
the area of the triangle is equal to half of the parallelogram.
Given: A ABC and a paralellogram BCDE on the same base BC and between the same parallels BC
and AD.
1
To prove: ar (ABC) = ar (parallelogram BCDE)
2
Construction: Draw AL ⊥ BC and DM ⊥ BC, meeting BC producted in M.
Proof: Since, E and D are colinear and BC || AD
 AL = DM ..............(i) [ distance between parallel lines is always same]

1
Now, ar (ABC) = (BC × AL)
2

1
 ar (ABC) = (BC × DM) [ AL = DM (from (i)]
2

1
 ar (ABC) = ar (parallelogam BCDE).
2
Theorem: The area of a trapezium is half the product of its height and the sum of the parallel
sides.

Given: Trapezium ABCD in which AB || DC, AL ⊥ DC, CN ⊥ AB and AL = CN = h (say), AB = a,


DC = b.
1
To prove: ar(trapezium ABCD) = h × (a + b).
2
Construction: Join AC.
Proof: AC is a diagonal of quad. ABCD.

 ar(trapezium ABCD) = ar(  ABC) + ar( ACD)


1 1 1
= h×a+ h×b= h(a + b). Hence Proved.
2 2 2
Theorem: Median of a triangle divides it into two triangles of equal area.
Given: A ABC in which AD is the median.
To Prove: ar (ABD) = ar(ADC).
Construction: Draw AL ⊥ BC.

PW OLYMPIAD WALLAH 50
IOQM
Proof: Since, AD is the median of ABC. Therefore, D is the midpoint of BC.

1
 ar (ABD) = (BD × AL) .. (i)
2
1
 ar (ADC) = (CD × AL)
2
1
 ar (ADC) = (BD × AL) .. (ii) [ BD = CD, AD is the median of ABC]
2
From (i) & (ii)
ar (ABD) = ar(ADC). Hence Proved.

Solved Examples

Example : In a parallelogram ABCD, AB = 8 cm. The altitudes corresponding to sides AB and AD are
respectively 4 cm and 5 cm. Find AD.
Solution : Area of a ||gm = Base × corresponding altitude
 Area of parallelogram ABCD = AD × BN = AB × DM
 AD × 5 = 8 × 4
8 4
 AD = = 6.4 cm.
5

Example : The diagonals of a parallelogram ABCD intersect in O. A line through O meets AB in X and the
1
opposite side CD in Y. Show that ar (quadrilateral AXYD) = ar(parallelogram ABCD).
2
Solution :  AC is a diagonal of the parallelogram ABCD.

1
ar ( ACD) = ar(|| gm ABCD) ...(i)
2
Now, in s AOX and COY,
AO = CO [Diagonals of a parallelogram bisect each other]
AOX = COY [Vertically opposite s]
OAX = OCY [Alternate interior s]
[ AB || DC and transversal AC intersects them]

 AOX  COY [By ASA congruency]


 ar(AOX) = ar(COY) ...(ii)
Adding ar(quad. AOYD) to both sides of (ii), we get
ar(quad. AOYD) + ar(AOX) = ar (quad. AOYD) + ar (COY)
1
 ar(quad. AXYD) = ar(  ACD) = ar(|| gm ABCD) Hence Proved.
2

PW OLYMPIAD WALLAH 51
IOQM
Example : ABCD is a trapezium with AB || DC. A line parallel to AC intersects AB at X and BC at Y. Prove
that ar(ADX) = ar(ACY).

Solution :

Join CX, DX and AY.


Clearly, triangles ADX and ACX are on the same base AX and between the parallels AB and DC.
 ar (ADX) = ar (ACX) ... (i)
Also,  ACX and  ACY are on the same base AC and between the parallels AC and XY.
 ar (ACX) = ar (ACY) ...(ii)
From (i) and (ii), we get
ar (ADX) = ar (ACY).

Example : ABCD is a quadrilateral. A line through D, parallel to AC, meets BC produced in P as shown in
figure. Prove that ar (ABP) = ar(quad. ABCD).

Solution : Since s ACP and ACD are on the base AC and between the same parallels AC and DP.
 ar(ACP) = ar(ACD)
 ar(ACP) + ar(ABC) = ar(ACD) + ar(ABC)
 ar(ABP) = ar(quad. ABCD).

Example : In figure, E is any point on median AD of a  ABC. Show that ar(  ABE) = ar(  ACE).

Solution : Construction: From A, draw AG ⊥ BC and from E draw EF ⊥ BC.


BD  AG DC  AG
Proof: ar( ABD) = and ar(  ADC) =
2 2
But, BD = DC [D is the mid-point of BC, AD being the median]

ar( ABD) = ar( ADC) ... (i)


BD  EF DC  EF
Again, ar( EBD) = and ar(  EDC) =
2 2
But, BD = DC
 ar( EBD) = ar( EDC) .. (ii)
Subtracting (ii) from (i), we get
ar( ABD) – ar( EBD) = ar(  ADC) – ar(  EDC)
 ar( ABE) = ar(  ACE). Hence Proved.

PW OLYMPIAD WALLAH 52
IOQM
Example : Triangles ABC and DBC are on the same base BC; with A, D on opposite sides of the line BC,
such that ar( ABC) = ar( DBC). Show that BC bisects AD.
Solution : Construction: Draw AL ⊥ BC and DM⊥ BC.
Proof: ar( ABC) = ar( DBC) [Given]
BC  AL BC  DM
 =
2 2
 AL = DM ...(i)
Now in  s OAL and OMD
AL = DM [From (i)]
 ALO = DMO [Each = 90º]
 AOL = MOD [Vertically opposite s]
 OLA  OMD [By AAS congruency]
 OA = OD [By CPCT]
i.e., BC bisects AD. Hence Proved.

Example : ABC is a triangle in which D is the mid-point of BC and E is the mid-point of AD. Prove that the
1
area of  BED = area of  ABC.
4
Solution : Given: A  ABC in which D is the mid-point of BC and E is the mid-point of AD.
1
To prove: ar(  BED) = ar(  ABC).
4
Proof:  AD is a median of  ABC.

1
 ar ( ABD) = ar( ADC) = ar (  ABC) ... (i)
2
[Median of a triangle divides it into two triangles of equal area]
Again,
 BE is a median of  ABD.

1
 ar ( BEA) = ar( BED) = ar (  ABD)
2
[Median of a triangle divides it into two triangles of equal area]

1 1 1
And ar( BED) = ar(  ABD) = × ar (  ABC) [From (i)]
2 2 2
1
 ar ( BED) = ar( ABC). Hence Proved.
4

Example : If the medians of a  ABC intersect at G, show that ar(  AGB) = ar(  AGC) = ar(  BGC)
1
= ar(  ABC).
3
Solution : Given: A  ABC and its medians AD, BE and CF intersect at G.
To prove:
1
ar( AGB) = ar(  AGC) = ar( BGC) = ar(  ABC).
3

PW OLYMPIAD WALLAH 53
IOQM
Proof: A median of a triangle divides it into two triangles of equal area.
In  ABC, AD is the median.
 ar( ABD) = ar( ACD) ...(i)
In GBC, GD is the median.
 ar(GBD) = ar(GCD) ...(ii)
Subtract equation (ii) from (i), we get
ar(ABD) - ar(GBD) = ar( ACD) - ar( GCD)
 ar(AGB) = ar(AGC) ..(iii)
Similarly, ar( AGB) = ar(  BGC) ..(iv)
From (iii) & (iv)
ar(AGB) = ar( AGC) = ar( BGC)
But, ar(ABC) = ar(  AGB) + ar(  AGC) + ar(  BGC) = 3 ar (  AGB)
1
 ar( AGB) = ar(  ABC).
3
1
Hence, ar( AGB) = ar( AGC) = ar( BGC) = ar(  ABC). Hence proved
3

Example : D, E and F are respectively the mid points of the sides BC, CA and AB of a  ABC. Show that:
(i) BDEF is a parallelogram
1
(ii) ar(||gmBDEF) = ar(ABC)
2
1
(iii) ar(DEF) = ar( ABC)
4

Solution :

(i) In  ABC,
 F is the mid-point of side AB and E is the mid point of side AC.

 EF || BD [Line joining the mid-points of any two sides of a  is parallel to the third side.]

Similarly, ED || FB.
Hence, BDEF is a parallelogram. Hence Proved.
(ii) Similarly, we can prove that AFDE and FDCE are parallelograms.
 FD is a diagonal of parallelogram BDEF.

 ar( FBD) = ar( DEF) ...(i)


Similarly, ar( FAE) = ar( DEF) ...(ii)
And, ar(DCE) = ar(DEF) ...(iii)

PW OLYMPIAD WALLAH 54
IOQM
From above equations, we have
ar( FBD) = ar(  FAE) = ar( DCE) = ar( DEF)
and ar ( FBD) + ar (DCE) + ar (DEF) + ar (FAE) = ar ( ABC)
 2 [ar( FBD)+ ar( DEF)] = ar(  ABC) [By using (ii) and (iii)]
1
 2 [ar. (IIgm BDEF)] = ar ( ABC)  ar (||gm BDEF) = ar ( ABC).
2
(iii) Since,  ABC is divided into four non-overlapping triangles FBD, FAE, DCE and DEF.
 ar( ABC) = ar(  FBD) + ar(  FAE) + ar( DCE) + ar( DEF)
 ar ( ABC) = 4 ar ( ABC) [Using (i), (ii) and (iii)]
1
 ar(DEF) = ar(ABC). Hence Proved.
4

Example : In figure, P is a point in the interior of a rectangle ABCD. Show that

1
(i) ar( APD) + ar( PBC) = ar(rectangle ABCD)
2
(ii) ar( APD) + ar( PBC) = ar(  APB) + ar(  PCD)
Solution : Construction: Draw EPF || AB || CD and LPM || AD || BC.
Proof:
(i) EPF || AB and DA cuts them.
 DEP = EAB = 90º [Corresponding angles]
 PE ⊥ AD.
Similarly, PF ⊥ BC; PL ⊥ AB and PM ⊥ DC.
1  1  1
 ar(APD) + ar(BPC) =   AD  PE  +   BC  PF  = AD × (PE+PF) [BC = AD]
2  2  2

1 1
= × AD × EF = × AD × AB [EF = AB]
2 2

1
= × ar(rectangle ABCD).
2

1  1  1
(ii) ar (APB) + ar(PCD) =   AB  PL  +   DC  PM  = × AB × (PL + PM) [ DC = AB]
2  2  2

1 1
= × AB × LM = × AB × AD [LM = AD]
2 2

1
= × ar(rect. ABCD).
2
 ar(  APD) + ar(PBC) = ar(  APB) + ar(PCD) Hence Proved.

PW OLYMPIAD WALLAH 55
IOQM
Example : ABCD is a parallelogram. X and Y are mid-points of BC and CD respectively.
3
Prove that area (AXY) = area (parallelogram ABCD).
8
Solution : Since X and Y are the mid–points of sides BC and CD respectively in BCD.
1
 XY || BD and XY = BD.
2
1
area (CYX) = area(DBC)
4
1
= area (parallelogram ABCD) .....(i)
8
1
[Since ar(DBC) = area (parallelogram ABCD)]
2
1
Parallelogram ABCD and ABX are between the same parallels AD and BC and BX = BC.
2
1
 area (ABX) = area (parallelogram ABCD) .....(ii)
4
1
Similarly, area (AYD) = area (parallelogram ABCD)
4
Now, ar(AXY) = ar(parallelogram ABCD) – [ar(ABX) + ar(AYD) + ar(CYX)]
1 1 1
ar(AXY) = ar (parallelogram ABCD) –  + +  ar(ABCD)
4 4 8

 5 3
ar(AXY) =  1 −  ar(parallelogram ABCD) = ar(parallelogram ABCD).
 8 8

Example : In the figure AE : EB = 1: 2 and BD : DC = 5 : 3. Find EG : GC.

Solution : Through E, we draw a line parallel to AD, intersecting BC at M.

2 1
BM = 5   and MD = 5  
3 3
5
EG : GC = MD : DC = : 3 = 5 : 9.
3

PW OLYMPIAD WALLAH 56
IOQM
MISCELLANEOUS ADVANCE SOLVED EXAMPLES:
Example : ABC is a triangle, BD is the- bisector of ABC and CD is
the bisector of ACX, DB is extended to Y. BE is the
bisector of YBC and DE is the bisector of BDC.
A = 60°, ABC = 80°. Calculate the angles of BDE
and BDC. Refer the given figure.
Solution : In ABC,
A + B + C = 180° [by angle sum property]
 C = 180° – 60°– 80° = 40°
By linear pair,
ACB + ACX = 180°
40° + ACX = 180°
 ACX = 140°
But CD bisects ACX.  ACD = DCX = 70°
BD bisects B = 80°
BCD = BCA + ACD = 110°
Now BDC = 180° – 40° – 110° = 30°
 The angles of BDC are 40°, 30° and 110°
By linear pair,
DBC + CBY = 180°
 40° + CBY = 180°
or CBY = 140°
But BE bisects CBY = 140°.
 EBY = EBC = 70°
DE bisects BDC = 30°
 BDE = 15°
DBE = DBC + CBE = 110°
Now,
BED = 180° – 110° – 15° = 55°
 The angles of BDE are 110°, 15° and 55°

Example : The perimeter of a triangle is 20. The lengths of three sides are all integers. How many triangle
are there are not congruent?
Solution : Method–1:
Let x be the length of the shortest side.
x = 2: (2, 9, 9) 1 triangle
x = 2: (3, 8, 9) 1 triangle
x = 2: (4, 7, 9), (4, 8, 8) 2 triangle
x = 2: (5, 6, 9), (5, 7, 8) 2 triangle
x = 2: (6, 6, 8), (6, 7, 7) 2 triangle
In total there are 8 such triangles.
Method – 2:
x = 9: (9, 9, 2), (9, 8, 3), (9, 7, 4), (9, 6, 5), (9, 6, 5) 4 triangle
x = 8: (8, 8, 4), (8, 7, 5), (8, 6, 6), (8, 5, 7), (8, 4, 8) 3 triangle
x = 7: (7, 7, 7) 1 triangle
In total there are 8 such triangles.

PW OLYMPIAD WALLAH 57
IOQM
Example : How many different isosceles triangles have integer side lengths and perimeter of 81 units?
1
Solution : Since, the longest side c < P and a + b > c
2

We have 14 + 6 = 20 different isosceles triangles.

Example : As shown in the figure, triangle ABC is divided into six smaller triangles by lines drawn from the
vertices through a common interior point. The areas of four of these triangles are as indicated.
Find the area of triangle ABC.
[CAP] CP [CBP]
Solution : = = yields
[FAP] FP [FBP]
84 + y x + 35
= , (14.1)
40 30
[CAP] AP [BAP]
and = = yields
[CD P] DP [BDP]

84 + y 70
= =2 (14.2)
x 35
(14.2) x x + 35
But , it follows that = ,
(14.1) 40 60

 3x = 2x + 70, i.e., x = 70. Then by (2), y = 140 – 84 = 56.


Thus, [ABC] = 84 + 56 + 40 + 30 + 35 + 70 = 315.

Example : Given an equilateral triangle, if the ratio of the area of its circumscribed circle to the area of its
inscribed circle is a : b (where a & bare coprime), then what is the value of a2 + b?
Solution : Given an equilateral triangle PQR. Let C1 be its circumscribed circle and C2 its inscribed circle.
Suppose QR, RP, PQ are tangent to C2 at P', Q', R', respectively. The area of triangle PQR is
4 times the area of triangle P'Q'R'. So the area of C1 is also 4 times the area of C2.

PW OLYMPIAD WALLAH 58
IOQM
Example : Consider an equilateral triangle ABC, where AB = BC = CA = 2011. Let P be a point inside
ABC. Draw line segments passing through P such that DE || BC, FG || CA and HI || AB.
− 122
Suppose DE : FG : HI = 8 : 7: 10. If DE+ FG +HI = , then find the value of .
150

Solution : Set DP= GP= a, IP= FP= b, EP= HP= c. Then


 = DE + FG +HI = (a + c) + (a + b) + (b + c) = 2 (a + b + c) = 2 × 2011 = 4022.

Example : In the figure below, ABC is a triangle, and D and E are points on AB and BC respectively. It is
given that DE is parallel to AC, and CE: EB = 1 : 3. If the area of ABC is 1440 cm2 and the area
of ADE is 3x cm2, what is the value of x?

Solution : The area of ABC is given to be S = 1440 cm2. Let S1 and S2 denote the areas of ADE and
DBE respectively. Since DE is parallel to AC, DBE and ABC are similar. Therefore
2 2
S2  BE   3 9
=  =  =
S  BC   4 16
9
Thus, S2 = S. As DE is parallel to AC, we have AD : DB = CE : EB = 1 : 3.
16
S1 1 1 1 9 3
Consequently, = . Hence S1 = S2 = . S= × 1440 = 270 cm2.
S2 3 3 3 16 16

Example : In the diagram below, the lengths of the three sides of the triangle are a cm, b cm and c cm. It is
a 2 + b2 cotC k−5
given that = 2011. If the value of is k, then find the value of .
c 2
cot A + cot B 100

Solution : By the laws of sine and cosine, we have


sin A sin B sin C a 2 + b2 − c2
= = and cos C = .
a b c 2ab

PW OLYMPIAD WALLAH 59
IOQM
Then
cot C cos C 1
= =
cot A + cot B sin C cos A sin B + cos B sin A
sin A sin B
sin A sin B cos C
=
sin(A + B) sin C

 sinA sin B 
=  cos C
 sin 2 C 

 (ab/ c 2 ) sin 2 C   a 2 + b 2 − c 2 
=  
 sin 2 C  2ab 

a 2 + b2 − c2
=
2c 2
2011 − 1
= = 1005
2

Example : In the diagram below, ABC is a triangle such that AB is longer than AC. The point N lies on BC
such that AN bisects BAC. The point G is the centroid of ABC, and it is given that GN is
perpendicular to BC. Suppose AC = 6 cm, BC = 5 3 cm and AB = x cm. Find the value of x.

Solution : Let BC = a, CA = b, and AB = c. We shall prove that c+ b = a 3 . Thus c = 5 3 × 3 – 6 = 9.


Using the angle bisector theorem, we have BN/NC = c/a > 1 so that BN > BM. Also ANC =
1 1
B + A < C + A < ANB so that ANC is acute. This shows that B, M, N, D are in
2 2
this order, where M is the midpoint of BC and D is the foot of the perpendicular from A onto BC.

a 2 + c2 − b2 ac
First we have BD = c cos B = . Using the angle bisector theorem, BN = .
2a b+c
a ac a a(c − b)
Therefore, MN = BN – = − = .
2 b+c 2 2(c + b)

a a 2 + c2 − b2 a c2 − b2
Also MD = BD – = − = .
2 2a 2 2a
Since GN is parallel to AD and G is the centroid of the triangle ABC, we have MD/MN = 3. If
follows that c + b = a 3 . Thus AB = a 3 – b = 15 – 6 = 9.
PW OLYMPIAD WALLAH 60
IOQM
Example : Let ABC be a non-isosceles acute-angled triangle with circumcentre O, orthocentre H and C = 41°.
Suppose of the bisector of A passes through the midpoint M of OH. Find HAO in degrees.
Solution : Let P be the midpoint of the arc BC not containing A on the circumcircle of the triangle ABC.
Then OP is the perpendicular bisector of BC. Since AM bisects A, the points A, M, P are
collinear. As both AH and OP are perpendicular to BC, they are parallel.
Thus HAM = OPM = OAM. Also HMA = OMP. Since HM =OM, we have the
triangles AHM and POM are congruent. Therefore AH = PO = AO.

Let L be the midpoint of BC . It is a known fact that AH = 20L. To see this, extend CO meeting the
circumcircle of the triangle ABC at the point N. Then AN BH is a parallelogram. Thus AH = NB =2OL.
Therefore in the right-angled triangle OLC, OC = OA = AH = 2OL. This implies OCL = 30°,
Since the triangle ABC is acute, the circumcentre O lies inside the triangle.
In fact A = 60° and B = 79°. Then OAC = OCA = 41° – 30° = 11°.
Consequently, HAO = 2OAM = 2 × (30° – 11°) = 38°.

Example : In the diagram below, A and B (20, 0) lie on the x–axis and C(0, 30) lies on the y–axis such that
ACB = 90°. A rectangle DEFG is inscribed in triangle ABC. Given that the area of triangle
x −8
CGF is 351, if the area of the rectangle DEFG is x, then find .
10

30 2 (20 + 45)  30
Solution : Note that AO = = 45. Then the area of ABC is = 975.
20 2
Let the height of CGF be h. Then
2 2
 h 351  3  h 3
  = =  
 
=
30 975 5 30 − h 2

2
Note that the rectangle DEFG has the same base as CGF. Then its area is 351 × × 2 = 468.
3

PW OLYMPIAD WALLAH 61
IOQM
Example : The lengths of the sides of a triangle are successive terms of a geometric progression. Let A and
C be the smallest and largest interior angles of the triangle respectively. If the shortest side has
sin A − 2 sin B + 3sin C 19
length 16 cm and = , then find the perimeter of the triangle in
sin C − 2 sin B + 3sin A 9
centimetres.
Solution : Let the lengths of the sides of the triangle in centimetres be 16, 16r and 16r2 (where r > 1)
1 − 2r + 3r 2 19 3  3 9
Then = so that r = . Hence, the perimeter of the triangle = 16  1 + +  = 76 cm
r − 2r + 3
2
9 2  2 4

Example : Given a triangle ABC, let B' and C' be points on the sides AB and AC such that. BB' = CC'. Let O
and O' be the circumcentres (i.e., the centre of the circumscribed circle) of ABC and AB'C' ,
1
respectively. Suppose OO' intersect lines AB' and AC' at B" and C", respectively. If AB = AC
2
and AB" = k AC", then find the value of k2.
Solution :

Let P and P' be the projections of O and O' on AB respectively. Then


1 1 1 1
PP' = AP – AP' = AB – AB' = (AB – AB') = BB'.
2 2 2 2
1
Similarly, let Q and Q' be the projections of O and O' on AC respectively, then QQ' = CC'.
2
PP ' QQ '
sin O'OP = = = sin O'OQ  O'OP = O'OQ.
OO ' OO '

So AB"C" = AC''B". it follows that AB" = AC".

Example : ABC is an equilateral triangle of side length 30. Fold the triangle so that A touches a point X on
k
BC. If BX = 6, find the value of . Where k is the length of the crease obtained from folding.
7

PW OLYMPIAD WALLAH 62
IOQM
Solution :

Apply the law of cosine on XBY and XCZ respectively:


p2 = 62 + (30 – p)2 – 6(30 – p),
q2 = 242 + (30 – q)2 – 24(30 – q).
Then p = 14 and q = 21. Applying the law of cosine in YXZ again to obtain
k = r2 = p2 + q2 – pq = 142 + 212 – 14 · 21 = 343.

Example : In the triangle ABC, AB = AC, A= 90°, D is the midpoint of BC, E is the midpoint of AC and F
is a point on AB such that BE intersects CF at P and B, D, P, F lie on a circle. Let AD intersect
CP at H. Given AP = 5 + 2, find the length of PH.

Solution :

Join PD. Then DPC = FBD = 45° = D AC so that D, P, A, Care concyclic.


Thus APC = ADC = 90°.
It follows that EA = EP =ED= EC. Let PAH = . Then PCD = .
Thus EPC = ECP = 45°–  so that AEB = 90° – 2.
That is ABE = 2. Thus tan 2 = AE/AB = 1/2.

From this, we get tan  = 5 – 2.

Therefore, PH = AP tan  = ( 5 + 2 )( 5 – 2) = 1.

PW OLYMPIAD WALLAH 63
IOQM

EXERCISE # 1

OBJECTIVE QUESTIONS
LINES AND ANGLES
1. In Figure, D and E are the mid–points of sides AB and AC respectively of ABC. Find EDB.

(A) 110º (B) 120º (C) 70º (D) 80º

2. In ABC, what is sum of the angles a + b + c + d + e + f + g + h + i ?

(A) 360º (B) 540º (C) 600º (D) Cannot be determined

3. P is a point inside ABC. If PBA = 20º BAC = 50º and, PCA = 35º, then the measure of BPC
is:

(A) 65º (B) 75º (C) 90º (D) 105º

4. In the given figure OBC and OKH are straight lines. If AH = AK, b = 80º and c = 30º then the value of d is:

(A) 20º (B) 25º (C) 30º (D) 45º

PW OLYMPIAD WALLAH 64
IOQM
5. In given figure find the values of x and y, if QS = RQ.

(A) x = 36º, y = 32º (B) x = 45º, y = 32º (C) x = 32º, y = 45º (D) x = 45º, y = 26º

CONGRUENT TRIANGLES & SIMILAR TRIANGLES


1. In a right angled triangle ABC, P is mid point of AC. Which one is true?
AC AC
(A) PA = (B) PB = (C) PA = PB (D) All of these
2 2

2. The sides of a triangle are in the ratio 4 : 6 : 11. Which of the following words best described the
triangle?
(A) obtuse (B) isosceles (C) acute (D) impossible

3. In the given diagram B = C = 65º and D = 30º, then the true statement is:

(A) BC = CA (B) CA > CD (C) BD > AD (D) AC = AD

4. In a PQR, PS is bisector of P, Q = 70º and R = 30º, then:


(A) QR < PR > PQ (B) QR > PR > PQ (C) QR = PR = PQ (D) QR < PR = PQ

5. In the right–angled triangle QPR given below, PS is the altitude to the hypotenuse. The figure is
followed by three possible inferences.

I. Triangle PQS and II. Triangle PSQ and


Triangle RPS are similar. Triangle RSP are congruent.
III. Triangle PSQ and
triangle RPQ are similar.
Mark the correct option
(A) I and II are correct (B) I and III are correct
(C) only II is correct (D) All three are correct

PW OLYMPIAD WALLAH 65
IOQM
6. In the given figure, ABC is a right angled triangle. Also FG || DE || BC and AG = GE = EB. If DE = 12
cm, then the measure of BC is:

(A) 12 cm (B) 18 cm (C) 24 cm (D) 30 cm

7. In the below diagram, ABCD is a rectangle with AE = EF = FB. What is the ratio of the areas of CEF
and that of the rectangle?

1 1 1
(A) (B) (C) (D) None of these
6 8 9

8. In the figure DF || AG, DE || AB, AB = 15, CD = 8, AD = x, DE = 10, FG = y and CG = 6. The ratio x :


y equal to :

(A) 1 : 2 (B) 1: 3 (C) 2 : 1 (D) 3 : 2

9. In the figure A = CED, CD = 8 cm CE = 10 cm, BE = 2cm, AB = 9cm, AD = b and DE = a. The


value of a + b is:

(A) 13 cm (B) 15 cm (C) 12 cm (D) 9 cm

PW OLYMPIAD WALLAH 66
IOQM
10. Let X be any point on the side BC of a triangle ABC. If XM, XN are drawn parallel to BA and CA
meeting CA, BA in M, N respectively ; MN meets CB produced in T. Then

(A) TB2 = TX  TC (B) TC2 = TB  TX (C) TX2 = TB  TC (D) TX2 = 2(TB  TC)

11. The altitude of the given triangle are AP = 3, BQ = 4 and CR = 5. What is the ratio of BC : AC : AB ?

(A) 20 : 15 : 12 (B) 20 : 12 : 15 (C) 15 : 12 : 20 (D) 3 :4 : 5

12. The perimeters of two similar triangles are 25 cm and 15 cm respectively. If one side of first triangle is
9 cm, then the corresponding side of the other triangle is:
(A) 6.2 cm (B) 3.4 cm (C) 5.4 cm (D) 8.4 cm

13. In a ABC, AB = AC = 2.5 cm, BC = 4 cm. Find its height from A to the opposite base:
(A) 1.5 cm (B) 1 cm (C) 2 cm (D) 3 cm

QR
14. Two triangles ABC and PQR are similar, if BC : CA : AB = 1 : 2 : 3, then is :
PR
2 1 1 2
(A) (B) (C) (D)
3 2 2 3

area(  DFE)
15. In the given figure, DE || BC and AD : DB = 5 : 4, find .
area(  CFB)

(A) 5 : 9 (B) 25 : 16 (C) 25 : 81 (D) None of these

PW OLYMPIAD WALLAH 67
IOQM
PYTHAGORAS THEOREM
1. In a triangle ABC, if B = 90º and D is the point in BC such that BD = 2 DC, then :
(A) AC2 = AD2 + 3 CD2 (B) AC2 = AD2 + 5 CD2
(C) AC2 = AD2 + 7 CD2 (D) AC2 = AB2 + 5 BD2

2. Let XOY be a right angled triangle with XOY = 90º. Let M and N be the midpoints of legs OX and
OY, respectively. Given that XN = 19 and YM = 22, the length XY is equal to
(A) 24 (B) 26 (C) 28 (D) 34

3. If CD = 15, DB = 9, AD bisects A, ABC = 90º, then AB has length:

(A) 32 (B) 18 (C) 7 (D) 24

4. If the sides of a right triangle are 9, 12 and 15 cm long, then the sum of squares of medians is:
(A) 227.5 (B) 337.5 (C) 537.5 (D) None of these

5. In the following figure, AE ⊥ BC, D is the mid point of BC, then x is equal to

1 2 a2  h+d c+d−h a 2 + b2 + d 2 − c2
(A)  b − d 2
−  (B) (C) (D)
a 4 3 2 4

QUADRILATERALS
1. In the figure, the area of square ABCD is 4 cm2 and E any point on AB. F, G, H and K are the mid point
of DE, CF, DG, and CH respectively. The area of KDC is –

1 1 1 1
(A) cm2 (B) cm2 (C) cm2 (D) cm2
4 8 16 32

PW OLYMPIAD WALLAH 68
IOQM
2. The line joining the mid points of the diagonals of a trapezium has length 3. If the longer base is 97,
then the shorter base is:
(A) 94 (B) 92 (C) 91 (D) 90

3. In the adjoining figure DP is parallel to AC, then the ratio of area of triangle PCB and quadrilateral
ABCD is

(A) 1 : 1 (B) 1 : 2 (C) 1 : 4 (D) 2 : 3

4. One side of a parallelogram has length 3, and another side has length 4. Let a and b denote the lengths
of the diagonals of the parallelogram. Which of the following quantities can be determined from the
given information?
I. a + b II a2 + b2 III a3 + b3
(A) Only I (B) Only II (C) Only III (D) Only I and II

5. In the trapezium shown, AB || DC, and E and F are the midpoints of the two diagonals. If DC = 60 and
EF = 5 then the length of AB is equal to:

(A) 40 (B) 45 (C) 50 (D) 55

EUCLID GEOMETRY
1. A proof is required for:
(A) Postulate (B) Axiom (C) Theorem (D) Definition

2. How many number of lines does pass through two distinct points.
(A) 1 (B) 2 (C) 3 (D) 4

3. Which of the following is an example of a geometrical line.


(A) Black Board (B) Sheet of paper
(C) Meeting place of two walls (D) Tip of the sharp pencil

4. Given four points such that no three of them are collinear, then the number of lines that can be drawn
through any of the two points is:
(A) 2 lines (B) 4 lines (C) 6 lines (D) 8 lines

5. Two planes intersect each other to form a:


(A) plane (B) point (C) straight line (D) angle

6. ‘Lines are parallel if they do not intersect’ is stated in the form of:
(A) an axiom (B) a definition (C) a postulate (D) a proof

PW OLYMPIAD WALLAH 69
IOQM
7. Select the wrong statement:
(A) Only one line can pass through a single point
(B) Only one line can pass through two distinct points
(C) A line consists of infinite number of points.
(D) If two circles are equal, then their radii are equal

8. Number of dimension(s) a surface has:


(A) 0 (B) 1 (C) 2 (D) 3

9. The number of line segments determined by three collinear points is:


(A) Two (B) Three (C) Only one (D) Four

10. If the point P lies in between M and N and C is midpoint of MP, then:
(A) MC + PN = MN (B) MP +CP = MN (C) MC +CN = MN (D) None of these

HERON’S FORMULA
1. All the 3 sides of a right triangle are integers and one side has a length 11 units. Area of the triangle in
square units lies between
(A) 1 and 100 (B) 100 and 200 (C) 200 and 300 (D) More than 300

2. In the figure given PM = 10 cm, MN = 15 cm and PN = 17 cm. Also QM = QX and XR = RN. Perimeter of
the PQR, is:

(A) 32 (B) 27 (C) 25 (D) 21

3. A triangle EFG is inscribed in a unit square ABCD with E on AB, F on DA, G on CD such that AE = DF =
CG = 1/3. The area of the triangle EFG is:
5 1 5 4
(A) (B) (C) (D)
18 3 9 9

4. A triangle of area 9 y cm2 has been drawn such that its area is equal to the area of an equilateral triangle of
side 6 cm. Then, the value of y is:
(A) 2 cm (B) 3 cm (C) 2 cm (D) 3 cm

5. A plot of land is in the shape of a right angled isosceles triangle. The length of the hypotenuse is 50 2 m.
The cost of fencing it at Rs. 3 per metre will be:
(A) less than Rs. 300 (B) less than Rs. 400 (C) more than Rs. 500 (D) more than Rs. 600

6. The perimeter of an isosceles triangle is equal to 14 cm, the lateral side is to the base in the ratio 5 : 4. The
area of the triangle is :
1 3
(A) 21 cm2 (B) 21 cm2 (C) 21 cm2 (D) 2 21 cm2
2 2

PW OLYMPIAD WALLAH 70
IOQM
7. In a trapezium ABCD with bases AB and CD, where AB = 52, BC = 12, CD = 39 and DA = 5. The area of
the trapezium ABCD, is:
(A) 182 (B) 195 (C) 210 (D) 260

8. ABCD is a rectangle with AB = 12 cm and BC = 7 cm. Point E is on AD with DE = 2 cm. Point P is on AB.
How far to the right of point. A should point P be placed so that the shaded area comprises exactly 40% of
the area of the rectangle?

(A) 8 (B) 8.4 (C) 8.2 (D) 8.6

9. Given an isosceles trapezium ABCD in order with AB = 6, CD = 12 and area 36 sq. units. Length of the side
BC is:
(A) 6 (B) 5 (C) 4.5 (D) 5.5

10. The length of the side of a rhombus is 10 units and its diagonals differ by 4. The area of the rhombus is:
(A) 108 (B) 96 (C) 84 (D) 48

PARALLELOGRAM
1. ABCD is a parallelogram. Points P and Q, on BC trisects it in three equal parts. PR and QS are also drawn
parallel to AB, then ar(APQ) = ........... ar(ABCD).

1 1 1 1
(A) (B) (C) (D)
3 2 4 6

2. In the figure, D and E are the mid-point of the sides AC and BC respectively of ABC. If ar(BED) = 12
cm2, then ar (AEC) =

(A) 48 cm2 (B) 24 cm2 (C) 36 cm2 (D) none of these

3. In  ABC, AD is a median and P is a point on AD such that AP : PD = 1 : 2, then the area of  ABP =
1 2
(A) × Area of  ABC (B) × Area of  ABC
2 3
1 1
(C) × Area of  ABC (D) × Area of  ABC
3 6

PW OLYMPIAD WALLAH 71
IOQM

EXERCISE # 2
1. In the figure, AB = BC = CD = DE = EF = FG = GA, then find DAE (approximately)

(A) 24º (B) 25º (C) 26º (D) None of these

2. If the sum of all angles except one of a convex polygon is 2180º, then the number of sides of the
polygon is
(A) 19 (B) 17 (C) 15 (D) 13

3. A square board side 10 centimeters, standing vertically, is tilted to the left so that the bottom–right
corner is raised 6 centimeters from the ground.

By what distance is the top–left corner lowered from its original position ?
(A) 1 cm (B) 2 cm (C) 3 cm (D) 0.5 cm

4. In the diagram given below, ABD = CDB = PQD = 90º. If AB : CD = 3 : 1, the ratio of CD : PQ is :

(A) 1 : 0.69 (B) 1 : 0.75 (C) 1 : 0.72 (D) None of the above.

5. In the figure below, AL is perpendicular to BC and CM is perpendicular to AB. If CL = AL = 2BL, find


MC/AM.

(A) 2 (B) 3 (C) 4 (D) Cannot be determined

PW OLYMPIAD WALLAH 72
IOQM
6. A square ABCD is constructed inside a triangle PQR having sides 10, 17 and 21 as shown in figure.
Find the perimeter of the square ABCD.

(A) 28 (B) 23.2 (C) 25.4 (D) 28.8

7. ABCD is a trapezium in which AB || CD. If ADC = 2ABC, AD = a cm and CD = b cm, then the
length (in cm) of AB is :
a 2 2
(A) + 2b (B) a + b (C) a+b (D) a + b
2 3 3

8. E is the midpoint of diagonal BD of a parallelogram ABCD. If the point E is joined to a point F on DA


1
such that DF = DA, then the ratio of the area of DEF to the area of quadrilateral ABEF is:
3
(A) 1 : 3 (B) 1: 4 (C) 1 : 5 (D) 2 : 5

9. The sides of rectangle are all produced in order, in such a way that the length of each side is increased
1
by 'k' times itself. The area of the new quadrilateral formed becomes 2 times the area of the original
2
rectangle. Find the value of 'k'.
1 5 3
(A) (B) (C) (D) None of these
2 2 2

10. In the figure, the quadrilateral ABCD is a rectangle, P lies on AD and Q on AB. The triangles PAQ,
QBC and PCD all have the same area, and BQ = 2. The length of AQ, is:
(A) 3 + 5 (B) 2 3 (C) 5 +1 (D) not uniquely determined

11. Find the angle between the two hands of a clock of 15 minutes past 4 O'clock
(A) 38.5° (B) 36.5° (C) 37.5° (D) None of these

12. At what time between 5 O'clock & 5 : 30 will the hands of a clock be at right angle ?
10 5
(A) 10 minutes past 5. (B) 11 minutes past 5.
11 11
10 9
(C) 9 minutes past 5. (D) 10 minutes past 5.
11 11

1 1
13. In figure, PA, QB and RC are each perpendicular to AC. Then : + =
x z

1
(A) 1 (B) y (C) (D) None
y

PW OLYMPIAD WALLAH 73
IOQM
14. In a triangle ABC, the internal bisector of the angle A meets BC at D. If AB = 4, AC = 3 and A = 60º,
then the length of AD is:
12 3 15 3
(A) 2 3 (B) (C) (D) None of these
7 8

15. In a triangle ABC, a straight line parallel to BC intersects AB and AC at point D and E respectively. If
the area of ADE is one–fifth of the area of ABC and BC = 10 cm, then DE equals:
(A) 2 cm (B) 2 5 cm (C) 4 cm (D) 4 5 cm

16. The side lengths of trapezium are 4 3 , 4 3 , 4 3 and 2 × 4


3 . Its area in the ratio of two relatively prime
positive integers, m and n. The value of (m + n) is equal to:
(A) 5 (B) 7 (C) 9 (D) 13

17. A rectangle is inscribed in a square creating four isosceles right triangle. If the total area of these four
triangles is 200. The length of the diagonal of the rectangle is:

(A) 10 (B) 15 (C) 20 (D) 25

18. The cost of leveling a rectangular ground at Rs.1.25 per sq. metre is Rs 900. If the length of the ground is 30
metres, then the width is:
(A) 330 metres (B) 34 metres (C) 24 metres (D) 18 metres

19. A rectangular lawn 60 metres by 40 metres has two roads each 5 metres wide running in the middle of it, one
parallel to length and the other parallel to breadth. The cost of gravelling the roads at 60 paise per sq. metre
is:
(A) Rs. 300 (B) Rs. 280 (C) Rs. 285 (D) Rs. 250

20. A child draws the figure of an aeroplane as given. Here the wings EDCF and AGHB are parallelograms, the
tail ADK is an isosceles triangle, the cockpit BLC is a semi-circle and the portion ABCD is a square. Let FP
⊥ CD & HQ ⊥ AB, AB = 6 cms. KD = 5 cms FP = HQ = 2 cms The area of the figure is:[ = 3.14]

(A) 86.14 cm2 (B) 87.25 cm2 (C) 84.63 cm2 (D) 91.56 cm2

21. The area of a rhombus is 28 cm2 and one of its diagonals in 4cm. Its perimeter is:
(A) 4 53 (B) 36 cm (C) 2 53 (D) 53 cm

22. The area of a trapezium shaped field is 960 m2 the distance between two parallel sides is 30 m. and one of
the parallel side is 20 m. Find the length of other parallel side.
(A) 44 m. (B) 22 m. (C) 88 m. (D) 11 m.

PW OLYMPIAD WALLAH 74
IOQM
23. E is the midpoint of diagonal BD of a parallelogram ABCD. If the point E is joined to a point F on DA such
1
that DF = DA, then the ratio of the area of DEF to the area of quadrilateral ABEF is:
3
(A) 1 : 3 (B) 1: 4 (C) 1 : 5 (D) 2 : 5

12
24. ABCD (in order) is a rectangle with AB = CD = and BC = DA = 5. Point P is taken on AD such that
5
BPC = 90º. The value of (BP + PC) is equal to:
(A) 5 (B) 6 (C) 7 (D) 8

MISCELLANEOUS ADVANCE PROBLEMS

25. In the adjoining figure ABC, DEF are equilateral triangles AB = 8 cm and DE = 3cm. Then the possible
value of AE + BD + CF is

(A) 6.9 cm (B) 7.1 cm (C) 5.2 cm (D) 8.3 cm

26. In the adjoining figure BAC is a 30° – 60° – 90° triangle with AB = 20. D is the midpoint of AC. The
perpendicular at D to AC meets the line parallel to AB through C at E. The line through E
perpendicular to DE meets BA produced at F. If DF = 5 x then find x.

(A) 1 (B) 7 (C) 5 (D) 2

27. In ABC, D, E, F are on the sides BC, CA, AB respectively, such that they are concurrent at a point G,
BD = 2CD. the areas S1 = [GEC] = 3, S2 = [GCD] = 4. Find the area of ABC.
(A) 30 (B) 20 (C) 25 (D) 35

28. In ABC, the bisector of A intersects BC at D. A perpendicular to AD from B intersects AD at E.


A line segment through E and parallel to AC intersects BC at G, and AB at H. If AB = 26, BC = 28,
AC = 30, find the measure of DG .

(A) 2 (B) 3 (C) 1 (D) 4

PW OLYMPIAD WALLAH 75
IOQM
29. The measures of the bases of trapezoid ABCD are 15 and 9, and the measure of the altitude is 4. Legs
DA and CB are extended to meet at E. If F is the midpoint of AD , and G is the midpoint of BC , find
the area of FGE. (The figure is not drawn to scale.)

(A) 48 (B) 49 (C) 46 (D) 52

30. In ABC, AB: AC = 4 : 3 and M is the midpoint of BC. E is a point on AB and F is a point on AC such
that AE : AF = 2 : 1. It is also given that EF and AM intersect at G with GF = 36 cm and GE = x cm.
And the value of x.

31. The diagram shows an equilateral triangle ADE inside a square ABCD. What is the value of
2
 area of ADE 
 area of DEC 

32. In the figure below, S is a point on OR and U is a point on PR. The line segments PS and QU intersect
at the point T. It is given that PT = TS and QS = 2RS. If the area of PQR is 150 cm2 and the area of
PSU is x cm2. Find the value of x.

PW OLYMPIAD WALLAH 76
IOQM
33. In the triangle ABC, AB = AC = 1, D and E are the midpoints of AB and AC respectively. Let P be a
point on DE and let the extensions of BP and CP meet the sides AC and AB at G and F respectively.
1 1
Find the value of + .
BF CG

34. In the triangle ABC, AB = 14, BC = 16, AC = 26, M is the midpoint of BC and D is the point on BC
such that AD bisects BAC. Let P be the foot of the perpendicular from B onto AD. Determine the
length of PM.

35. Triangle ABC is an isosceles right triangle with AB = AC = 3. Let M be the midpoint of hypotenuse
BC . Points I and E lie on sides AC and AB , respectively, so that AI > AE and AIME is a cyclic
a− b
quadrilateral. Given that triangle EMI has area 2, the length CI can be written as , where a, b
c
and c are positive integers and b is not divisible by the square of any prime. What is the value of a + b + c?

36. In ABC shown in the figure, AB = 7, BC = 8, CA = 9, and AH is an altitude. Point D and E lie on
sides AC and AB , respectively, so that BD and CE are angle bisectors, intersecting AH at Q and P,
a
respectively. If PQ = c . where a, b are co–prime, then what is the value of (a + b + c2) ?
b

37. Triangle ABC has AB = 27, AC = 26, and BC = 25. Let I denote the intersection of the internal angle
bisectors of ABC. What is BI?

38. In triangle ABC, AB = (b2 – 1) cm, BC = a2 cm and AC = 2a cm, where a and b are positive integers
greater than 1. And the value of a – b.

PW OLYMPIAD WALLAH 77
IOQM
39. In the triangle ABC, the bisectors of A and B meet at the incentre I, the extension of AI meets the
circumcircle of triangle ABC at D. Let P be the foot of the perpendicular from B onto AD, and Q a
point on the extension of AD such that ID = DQ. Determine the value of (BQ × lB)/(BP × ID).

40. Three sides OAB, OAC and OBC of a tetrahedron OABC are right-angled triangles, i.e. AOB =
AOC= BOC = 90°. Given that OA = 7, OB = 2 and OC = 6, find the value of
[(Area of AB) 2 +(Area of OAC)2 +(Area of OBC)2 +(Area of ABC)2 + 37]1/2.

41. A triangle is partitioned into three triangles and a quadrilateral by drawing two lines from vertices to
their opposite sides. The areas of the three triangles are 3, 7, and 7, as shown. What is the area of the
shaded quadrilateral?

42. In ABC point D and E lie on BC and AC , respectively. If AD and BE intersect at T then AT/DT =
 11CD 
3 cm and BT/ET = 4. What is  ?
 BD 

43. Triangle ABC has a right angle at C, AC = 3 and BC = 4. Triangle ABD has right angle at A and AD =
12. Points C and D are on opposite sides of AB . The line through D parallel to AC meets CB
DE m m+n
extended at E. If = where m and n are relatively prime positive integers, then is
DB n 2

PW OLYMPIAD WALLAH 78
IOQM
44. Equilateral triangle ABC has been creased and folded so that vertex A now rests at A' on BC as shown.
k 21
If BA' = 1 and A'C = 2. If the length of crease PQ is , find k.
20

45. Vertex E of equilateral triangle ABE is in the interior of square ABCD, and F is the point of

intersection of diagonal BD and line segment AE. If length AB is 1 + 3 and the area of ABF is
given by A. Find (4A)2.

46. A ray of light originates from point A and travels in a plane, being reflected n times between lines AD
and CD, before striking a point B (which may be on AD or CD) perpendicularly and retracing its path
to A. (At each point of reflection the light makes two equal angles as indicated In the adjoining figure.
The figure shows the light path for n = 3.) If CDA = 8°, what is the largest value n can have?

47. A thin piece of wood of uniform density in the shape of an equilateral triangle. with side length 3 inches
weighs 12 gm. A second piece of the same type of wood, with the same thickness, also in the shape of
an equilateral triangle, has side length of 5 inches. If W is the weight in gm, of the second piece. Then
find [W], where [x] denotes the greatest integer less than or equal to x.

48. The isosceles right triangle ABC has right angle at C and area 12.5 The rays trisecting ACB intersect
a(2 − b )
AB at D and E. If the area of CDE can be written as . Find the product of a and b.
2

PW OLYMPIAD WALLAH 79
IOQM

EXERCISE # 3

IJSO STAGE-I (PREVIOUS YEARS)


1. L, M and N are mid points of sides AB, BC and CA of triangle ABC. If area of triangle ABC is 48
units, the area of triangle LMN will be: [IJSO–2008]
(A) 6 units (B) 8 units (C) 12 units (D) 24 units

2. The adjacent sides of a parallelogram are 15 cm and 10cm. If length of one diagonal of this
parallelogram is 20 cm, the length of other diagonal will be: [IJSO–2008]
(A) 30 cm (B) 5 10 (C) 10 5 cm (D) 4 30 cm

3. In the parallelogram, the value of 'x' is: [NSTSE–2009]

(A) 30º (B) 50º (C) 70º (D) 80º

4. Given two similar triangles one of which has twice the perimeter of the other, by what factor is the area
of the larger triangle bigger than the smaller? [NSTSE–2009]
(A) 2 (B) 4 (C) 2 (D) 2 2

5. In the figure shown, PQ || RS and SM || TN. Then measure of angle  is: [IJSO–2009]

(A) 58º (B) 118º (C) 89º (D) 91º

6. The diagonals of a quadrilateral are equal and bisect each other. The quadrilateral has to be:
[IJSO–2009]
(A) any parallelogram (B) any rectangle (C) any trapezium (D) any rhombus

1
7. ABCD is a parallelogram. 'P' is a point on AD such that AP = AD and 'Q' is a point on BC such that
3
1
CQ = BC. Then AQCP is a: [NSTSE–2010]
3
(A) Parallelogram (B) Rhombus (C) Rectangle (D) Square

PW OLYMPIAD WALLAH 80
IOQM
8. The degree measure of each of the three angles of a triangle is an integer. Which of the following could
not be the ratio of their measures? [NSTSE–2010]
(A) 2: 3 : 4 (B) 3 : 4 : 5 (C) 5 : 6 : 7 (D) 6: 7 : 8

9. ABC is a right angled triangle, where B = 90º. CD and AE are medians. If AE = x and CD = y then,
correct statement is: [NSTSE–2010]

3 5
(A) x2 + y2 = AC2 (B) x2 + y2 = 2AC2 (C) x2 + y2 = AC2 (D) x2 + y2 = AC2
2 4
10. If the hypotenuse of a right angled triangle is 41 cm and the area of the triangle is 180 sq cm, then the
difference between the lengths of the legs of the triangle must be: [NSTSE 2010]
(A) 22 cm (B) 25 cm (C) 27 cm (D) 31 cm

11. The sides of a triangle are in the ratio 6 : 8 : 9 . Therefore: [IJSO–2010]


(A) Angles of the triangle are in the ratio 6 : 8 : 9
(B) It is an acute angled triangle
(C) It is a right angled triangle
(D) It is an obtuse angled triangle

12. In the figure shown AB is parallel to DE. The difference between angles x and y is: [IJSO–2010]

(A) 0º (B) 4º (C) 10º (D) 12º

13. AD, BE and CF are the medians of ABC. The sum of lengths of segments BE and CF is:
[IJSO–2010]
3 5 3 2
(A) < BC (B) > BC (C) > BC (D) < BC
2 3 2 3

14. All the three sides of a ABC have lengths in integral units, with AB = 2001 units and BC = 1002 =
units. The possible number of triangles with this condition is: [IJSO–2010]
(A) 2001 (B) 2002 (C) 2003 (D) 2004

PW OLYMPIAD WALLAH 81
IOQM
15. If the altitudes of a triangle are in the ratio 2 : 3 : 4, then the lengths of the corresponding sides are in
the ratio: [IJSO–2010]
(A) 2 : 3 : 4 (B) 6 : 4 : 3 (C) 3 : 2 : 4 (D) 3 : 2 : 1

16. In the figure shown below, DE || BC and AD = 3x – 2, AE = 5x – 4, BD = 7x – 5 and CE = 5x – 3.


Therefore, the value of x is [IJSO–2011]

7 7 10
(A) only 1 (B) only (C) 1 or (D)
10 10 7

17. In  ABC, D is a point on side AC such that ABD = ½ ABC. If AB = 36, BC = 48, CD = 28, then
the length DA will be [IJSO–2011]
(A) 20 (B) 21 (C) 22 (D) 24

18. Consider triangles having integer sides such that no side is greater than 4 units. How many such
triangles are possible? [IJSO–2012]
(A) 13 (B) 17 (C) 24 (D) 64

19. In the adjoining figure AQ = 2, QB = 4, BP = 3, PC = 5, CR = 6 and RA = 4. Find the area of triangle


PQR.

(A) 4.8 (B) 5.2 (C) 5.8 (D) 6.2

20. Let ABC be a triangle in which AB = AC. Let D be a point on BC such that AD bisects angle A. Value
BD
of the ratio is [IJSO–2013]
DC
(A) 1 : 1 (B) 1: 2 (C) 2 : 1 (D) none of these

21. Let m be the number of distinct (non congruent) integer–sided triangles each with perimeter 15 and n be
the number of distinct (non congruent) integer sided triangles each with perimeter 16. Then m– n equals
[IJSO–2013]
(A) –2 (B) 0 (C) 2 (D) – 4

PW OLYMPIAD WALLAH 82
IOQM
22. At what time (to the nearest second) immediately after 4 O' clock will angle between the hands of the
clock be the same as that at 4 O' clock? [IJSO–2013]
h m s h m s h m s h m s
(A) 4 42 50 (B) 4 43 38 (C) 4 43 40 (D) 5 5 27

23. A ray of light is incident on system of mirror as shown in the adjacent figure. What is the total
deflection (d) of the ray when it emerges out after two reflections? [IJSO–2014]

(A) 220° (B) 180° (C) 120° (D) 140°

24. If the distance between A and B is 230 km, B and C is 120 km, C and A is 350 km. Also, if the distance
between C and D is 200 km, distance between D and B is 330 km and distance from A to E is 100 km
and distance between D and E is 570 km. The diagram (not drawn to scale) that represents this
graphically is: [IJSO–2014]

(A) (B) (C) (D) none of these

25. The adjacent sides of a parallelogram are 30 cm and 20 cm. The length of one of the diagonal is 40 cm.
What is the length of the other diagonal? [IJSO–2015]
(A) 60 cm (B) 10 10 cm (C) 20 5 cm (D) 8 30 cm

26. In the adjoining figure segment AD, BE and CF are the altitudes of triangle ABC. Find AD × BC if
AB × AC = 409.6, BE × CF = 202.5. [IJSO–2015]

(A) 225 (B) 256 (C) 288 (D) 312

27. The angle between the hour arm and the minute arm of a clock at 2:10 a.m. is [IJSO–2016]
(A) zero (B) 4º (C) 5º (D) 6º

PW OLYMPIAD WALLAH 83
IOQM
28. If ABCD is a rhombus and  ABC = 60 º then [IJSO–2016]
(A) The point A, B, C, D are concylic
(B) The quadrilateral has exactly half the area of the square with same sides as ABCD
3
(C) The quadrilateral has area AB2
2
(D) The diagonals of the quadrilateral ABCD are equal and bisect each other at right angle

29. Diagonals of a quadrilateral bisect each other. Therefore the quadrilateral must be a [IJSO 2017]
(A) rectangle (B) rhombus (C) parallelogram (D) square

30. What is the radius of the circumcircle of a triangle whose sides are 30 cm, 36 cm and 30 cm.
[IJSO 2017]
(A) 15 cm (B) 16 cm (C) 17 cm (D) 18.75 cm

31. In triangle ABC, segment AD, segment BE and segment CF are altitudes. If AB × AC = 172.8 cm2 and
BE × CF = 108.3 cm2 then AD × BC = [IJSO 2017]
(A) 136.8 cm2 (B) 132.4 cm2 (C) 129.2 cm2 (D) 128.6.cm2

32. In a square ABCD, a point P is inside the square such that ABP is an equilateral triangle. The segment
AP cuts the diagonal BD in E. Suppose AE = 2. The area of ABCD is [IJSO (Stage-I) – 2018]
(A) 4 + 2 3 (B) 5 + 2 3 (C) 4 + 4 3 (D) 5 + 4 3

33. For a regular k-sided polygon, let (k) denotes its interior angle. Suppose n > 4 is such that (n-2),
(n), (n+3) forms an arithmetic progression. The sum of digits of n is – [IJSO (Stage-I) – 2018]
(A) 2 (B) 3 (C) 4 (D) 5

34. In a triangle ABC, let AD be the median from A; let E be a point on AD such that AE : ED = 1 : 2; and
let BE extended meets AC in F. The ratio of AF/FC is [IJSO (Stage-I) – 2018]
(A) 1/6 (B) 1/5 (C) 1/4 (D) 1/3

35. In a triangle ABC, a point D on AB is such that AD : AB = 1 : 4 and DE is parallel to BC with E on


AC. Let M and N be the mid points of DE and BC respectively. What is the ratio of the area of the
quadrilateral BNMD to that of triangle ABC? [IJSO (Stage-I) – 2018]
(A) 1/4 (B) 9/32 (C) 7/32 (D) 15/32

SUBJECTIVE QUESTIONS
36. In a rectangle ABCD, E is the midpoint of AB; F is a point on AC such that BF is perpendicular to AC;
and FE perpendicular to BD. Suppose BC = 8 3 . Find AB. [PRMO-2017]

37. Let P be an interior point of a triangle ABC whose side length s are 26, 65, 78. The line through P
parallel to BC meets AB in K and AC in L. The line through P parallel to CA meets BC in M and BA in
N. The line through P parallel to AB meets CA in S and CB in T. If KL, MN, ST are of equal lengths,
find this common length. [PRMO-2017]

PW OLYMPIAD WALLAH 84
IOQM
38. Let ABCD be a rectangle and let E and F be point son CD and BC respectively such that are (ADE) = 16,
area (CEF) = 9 and area (ABF) = 25. What is the area of triangle AEF ? [PRMO 2017]

39. Let AB and CD be two parallel chords in a circle with radius 5 such that the centre O lies between these
chords. Suppose AB = 6, CD = 8. Suppose further that the area of the part of the circle lying between the
chords AB and CD is (m + n) / k, where m, n, k are positive integers with gcd (m, n, k) = 1. What is the
value of m + n + k? [PRMO 2017]

40. Let 1 be a circle with centre O and let AB be a diameter of 1. Let P be a point on the segment OB
different from O. Suppose another circle is 2 with centre P lies in the interior of 1. Tangents are drawn
from A and B to the circle 2 intersecting 1 again at A1 and B1 respectively such that A1 and B1 are on
the opposite sides of AB. Given that A1B = 5, AB1 = 15 and OP = 10, find the radius of is 1.
[PRMO 2017]

41. In a quadrilateral ABCD, it is given that AB = AD = 13, BC = CD = 20, BD = 24. If r is the radius of the
circle inscribable in the quadrilateral, then what is the integer closest to r? [PRMO-2018]

42. Let ABCD be a trapezium in which AB || CD and AD ⊥ AB. Suppose ABCD has an incircle which
touches AB at Q and CD at P. Given that PC = 36 and QB = 49, find PQ. [PRMO-2018]

43. Let AB be a chord of a circle with centre O. Let C be a point on the circle such that ABC = 30º and O
lies inside the triangle ABC. Let D be a point on AB such that DCO = OCB = 20°. Find the measure
of CDO in degrees. [PRMO-2018]

44. In a triangle ABC, the median from B to CA is perpendicular to the median from C to AB. If the median
from A to BC is 30, determine (BC2 + CA2 + AB2)/100. [PRMO-2018]

45. In a triangle ABC, right-angled at A, the altitude through A and the internal bisector of A have lengths
3 and 4, respectively. Find the length of the median through A. [PRMO-2018]

46. Triangles ABC and DEF are such that A = D, AB = DE = 17, BC = EF = 10 and AC – DF = 12. What
is AC + DF [PRMO-2018]

47. Let ABC be an acute-angled triangle and let H be its orthocentre. Let G1, G2 and G3 be the centroids of
the triangles HBC, HCA and HAB respectively. If the area of triangle G1 G2 G3 is 7 units, what is the area
of triangle ABC? [PRMO-2018]

48. Let D be an interior point of the side BC of a triangle ABC. Let I1 and I2 be the incentres of triangles
ABD and ACD respectively. Let AI1 and AI2 meet BC in E and F respectively. If  BI1 E = 60°, what is
the measure of CI2 F in degrees? [PRMO-2018]

PW OLYMPIAD WALLAH 85
IOQM
49. From a square with sides of length 5, triangular pieces from the four corners are removed to form a
regular octagon. Find the area removed to the nearest integer? [PRMO 2019]

50. Let ABC be a triangle and let be  its circumcircle. The internal bisectors of angles A, B and C intersect
 at A1, B1 and C1 respectively and the internal bisectors of angles A1, B1 and C1 of the triangle A1B1C1
intersect  at A2, B2 and C2, respectively. If the smallest angle of triangle ABC is 40º. What is the
magnitude of the smallest angle of triangle A2B2C2 in degrees? [PRMO 2019]

51. Let AB be a diameter of a circle and let C be a point on the segment AB such that AC : CB = 6 : 7. Let D
be a point on the circle such that DC is perpendicular to AB. Let DE be the diameter through D. If [XYZ]
denotes the area of the triangle XYZ, find ABD / CDE to the nearest integer. [PRMO 2019]

52. Let ABCD be a convex cyclic quadrilateral. Suppose P is a point in the plane of the quadrilateral such
that the sum of its distances from the vertices of ABCD is the least. If {PA, PB, PC, PD} = {3, 4, 6, 8},
What is the maximum possible area of ABCD? [PRMO 2019]

53. A village has circular wall around it, and the wall has four gates pointing north, south, east and west. A
tree stands outside the village, 16 m north of the north gate and it can be just seen appearing on the
horizon from a point 48 m east of the south gate. What is the diameter in meters of the wall that
surrounds the village? [PRMO 2019]

54. Let ABC be a triangle with sides 51, 52, 53. Let  denote the incircle of ABC. Draw tangents to 
which are parallel to the sides of ABC. Let r1, r2, r3 be the inradii of the three corner triangles so formed.
Find the largest integer that does not exceed r1 + r2 + r3. [PRMO 2019]

55. In triangle ABC, the median AD (with D on BC) and the angle bisector BE (with E on AC) are
perpendicular to each other. If AD = 7 and BE = 9, find the integer nearest to the area of triangle ABC.
[PRMO 2019]

1
56. Let ABC be a triangle with AB = AC. Let D be a point on the segment BC such that BD = 48 and
61
DC = 61. Let E be a point on AD such that CE is perpendicular to AD and DE = 11. Find AE.
[IOQM 2021]

PW OLYMPIAD WALLAH 86
IOQM

Answer Key

EXERCISE # 1

OBJECTIVE QUESTIONS

LINES AND ANGLES


1. (A) 2. (B) 3. (D) 4. (B) 5. (D)

CONGRUENT TRIANGLES & SIMILAR TRIANGLES


1. (D) 2. (D) 3. (C) 4. (B) 5. (B)
6. (B) 7. (A) 8. (C) 9. (A) 10. (C)
11. (A) 12. (C) 13. (A) 14. (B) 15. (C)

PYTHAGORAS THEOREM
1. (B) 2. (B) 3. (B) 4. (B) 5. (A)

QUADRILATERALS
1. (B) 2. (C) 3. (A) 4. (B) 5. (C)

EUCLID GEOMETRY
1. (C) 2. (A) 3. (C) 4. (C) 5. (C)
6. (B) 7. (A) 8. (C) 9. (B) 10. (C)

HERON’S FORMULA
1. (D) 2. (B) 3. (A) 4. (B) 5. (C)
6. (D) 7. (C) 8. (B) 9. (B) 10. (B)

PARALLELOGRAM
1. (D) 2. (B) 3. (D)

EXERCISE # 2

Ques. 1 2 3 4 5 6 7 8 9 10 11 12 13 14 15 16 17 18 19 20
Ans. C C B B B B B C A C C A C B B D C C C A
Ques. 21 22 23 24 25 26 27 28 29
Ans. A A C C D B A C A

30. 54 31. 03 32. 20 33. 03 34. 06 35. 12 36. 48


37. 15 38. 0 39. 02 40. 33 41. 18 42. 04 43. 64
44. 07 45. 12 46. 10 47. 33 48. 75

PW OLYMPIAD WALLAH 87
IOQM

EXERCISE # 3
Ques. 1 2 3 4 5 6 7 8 9 10 11 12 13 14 15 16 17 18 19 20
Ans. C B C B C B A D D D B B C C B A B A C A
Ques. 21 22 23 24 25 26 27 28 29 30 31 32 33 34 35 36 37 38 39 40
Ans. C B D D B C C C C D A A B C D 24 30 30 75 20
Ques. 41 42 43 44 45 46 47 48 49 50 51 52 53 54 55 56
Ans. 8 84 80 24 24 30 63 30 4 55 13 55 48 15 47 25

PW Web/App - https://smart.link/7wwosivoicgd4

Library- https://smart.link/sdfez8ejd80if

PW OLYMPIAD WALLAH 88

You might also like